Sei sulla pagina 1di 67

in CIVIL LAW

Prepared by: Atty. Roney Jone P. Gandeza


event of my
operation.

REVIEW NOTES IN

CIVIL LAW

1. Abe is to be operated on for a


cancerous tumor. Before the
operation,
he
delivers
an
envelope to his friend, Rey. The
envelope
contains
a
letter
saying, I realize my days are
numbered, and I want to give
you this check for P1 million in
the event of my death from this
operation. Rey
cashes
the
check.
The surgeon performs the
operation
and
removes
the
tumor.
Abe
recovers
fully.
Several months later, Abe dies
from a heart attack that is
totally
unrelated
to
the
operation. Upon Abes death,
the administrator of his estate
immediately
commences
an
action to recover from Rey the
P1 million.
Will the suit prosper?
Yes. The donation to Rey is a
donation mortis causa, not inter
vivos. It is a mortis causa donation
because Abe intended it to take
effect upon his death, i.e. in the

from

this

That Abe died from a cause


unrelated to the operation does not
detract from the fact that Abes
death is the operative cause that
would have conveyed ownership of
the Php1 million given to Rey. Since
the donation was not expressed in
accordance with the formalities of
wills, the donation is void.

ATTY. RONEY JONE P. GANDEZA


UNIT 10 2nd FLOOR BBCCC BLDG.
NO. 56 COOPERATIVE ST. COR.
ASSUMPTION RD.
BAGUIO CITY
__________
LAW PROFESSOR
COLLEGE OF LAW
UNIVERSITY OF THE CORDILLERAS
HARRISON ROAD, BAGUIO CITY
__________

death

2. Rey
drafted
and
properly
executed a notarial will. Assume
the following clauses in his will
and the following events:
(a)Reys will provides, I leave my
2011 white Mitsubishi Montero
with Plate No. AFB 346 to my
friend Abe. At the time of Reys
death, Abe has already died,
leaving one child, Zandro.
(b)Reys will provides, I leave to
my friend, Abe, my 2011 white
Mitsubishi Montero with Plate
No. AFB 346. Just prior to Reys
death, he sold the Montero.
Discuss fully each situation
and the legal effect of the
events on the testamentary
provisions.
(a) The bequest to Abe is rendered void
by Abes predecease. The property
devolves to Reys heirs in intestacy.
Zandro, Abes son, has no right to
the property because a voluntary
heir, such as Abe, who dies before
the testator transmits nothing to his
heirs. (par. 1, Art. 856, CC)
(b)
The sale by Rey of
property
bequeathed
to
constitutes a revocation of
bequest in favor of Abe. The

I press toward
BAR OPERATIONS
2013 the mark for the

prize of the high calling of God


in Christ Jesus. - Philippians
3:14

BARRISTERS CLUB

Page 1 of 67

the
Abe
the
law

in CIVIL LAW
Prepared by: Atty. Roney Jone P. Gandeza
grants Rey this right because of the
ambulatory character of a will. Abe
has no right to question the sale
because the right of an instituted
heir (legatee or devisee) is merely
inchoate.

psychologically incapacitated to
enter
into
marriage
under
Philippine law. Under French law,
the marriage is voidable. Is the
marriage also voidable in the
Philippines?

3.
Debtor
owed
Creditor
P400, 000. The debt is the
subject of a lawsuit, and the
court
awards
Creditor
a
judgment of P400, 000 against
Debtor. To satisfy the judgment,
the sheriff levies on Debtors
family home in Baguio City
valued at P500, 000. Debtor
opposes
the
levy
on
the
allegation that the family home
is
exempt
from
execution.
Judgment for whom?

The marriage is void. The rule


is: valid there, valid here; void
there, void here; voidable there,
voidable here. However, the case
falls under one of the exceptions,
Article 36 of the Family Code which
refers to a void marriage by reason
of the psychological incapacity of a
spouse.
5.
What is an agency coupled
with an interest?
An agency coupled with an
interest is a relationship created for
the benefit of the agent. The agent
actually acquires a beneficial interest
in the subject matter of the agency.
Under these circumstances, it is not
equitable to permit a principal to
terminate the agency at will. Hence,
this type of agency is irrevocable.

Judgment for Creditor. Under


Article 160 of the Family Code, if
judgment is rendered against the
owner or owners of a family home,
and the creditor has reasonable
ground to believe that the value of
the family home is in excess of P300,
000 or P200, 000, as the case may
be, the creditor may apply for an
order directing the sale of the family
home.

6.
Aristotle needs P100, 000.
Socrates agrees to lend him the
money, but not without security.
Consequently, Aristotle delivers
some of his jewelry to Socrates
and signs a power of attorney
giving Socrates the power, in
case he fails to repay the loan,
to sell the jewelry as his agent
for the best price that can be
obtained and to pay out of the
proceeds the unpaid amount of
the loan, giving any surplus to
him. Having obtained the money,
Aristotle later tells Socrates that
he revokes the power to sell.

In the case presented, the


value of the Debtors family home is
in excess of P300, 000, it being
located in an urban area; hence, the
same may be sold at public auction
to satisfy the judgment against him.
The foregoing rule applies
even if the increase in value of
Debtors family home resulted from
improvements introduced by Debtor.
4.
Baldomero,
a
Filipino,
marries Antonette, his French
girlfriend, in Paris. At the time of
the marriage, Baldomero is

Is the agency revoked?

I press toward
BAR OPERATIONS
2013 the mark for the

prize of the high calling of God


in Christ Jesus. - Philippians
3:14

BARRISTERS CLUB

Page 2 of 67

in CIVIL LAW
Prepared by: Atty. Roney Jone P. Gandeza
No. Under the law of agency,
the power is not revoked. Aristotle
has no right to terminate the agency
at will. This is because the agency is
coupled with an interest. If Aristotle
dies, the power is still not affected.

sued for breach of


Judgment for whom?

contract.

Judgment for Seller. Buyers


telegram of December 16, refusing
to the terms stated in Sellers letter
of December 8, varied the number of
computer units, and was therefore a
counter-offer. A counter-offer is in
law a rejection of the original offer.
On December 8, the Seller declined
to fulfill the Buyers order, thus the
negotiations between the two parties
was closed. The Buyers attempt to
fall back on the Sellers original offer
by the telegram of December 19,
therefore, created no rights against
the Seller.
9.
A and B, Filipinos, both
women, enter into a same-sex
marriage in Denmark, where
such a marriage is valid. Is the
marriage also valid here?

7.
A and B orally contract for
the sale of As house for P1
million. A writes B a letter
confirming
the
sale
by
identifying the parties and the
essential terms description and
location of the property, price,
place of payment, and method of
payment and signs the letter.
Is the sale enforceable by
court action?
No. the sale was made orally.
However, A has made a written
memorandum of the oral land
contract. Because A signed the
letter, he can be held to the oral
contract by B. Because B has not
signed
a
written
contract
or
memorandum, B can plead the
Statute of Frauds as a defense, and A
cannot enforce the contract against
him.

The general rule enunciated in


the first paragraph of Article 26 of
the Family Code should apply, under
which
marriages
valid
where
celebrated are also valid here. The
case does not fit into any of the
exceptions
enunciated
in
the
foregoing provision of Article 26 of
the Family Code. Therefore, same
sex marriage is valid here if valid
where celebrated.

8.
Seller, in reply to an
inquiry from Buyer, sent a letter
dated December 8 stating terms
upon which he would sell 100 to
300 computer units of a certain
brand at a certain price. On
December 16, Buyer sent a
telegram to Seller ordering 90
computer units on those terms.
On December 18, Seller sent a
telegram to Buyer rejecting the
order. The next day Buyer sent
Seller
a
telegram
stating,
Please enter an order for 150
computer units as per your
letter of December 8. Seller
refused the order, and Buyer

However, it would seem that


the case falls under one of the
exceptions to the application of the
proper foreign law i.e., it runs
counter to an important public policy
of the forum marriage should be
between a man and a woman.
Because of this important public
policy, same sex marriage is void
here.
10.
A politician promises a
newspaper editor that he will

I press toward
BAR OPERATIONS
2013 the mark for the

prize of the high calling of God


in Christ Jesus. - Philippians
3:14

BARRISTERS CLUB

Page 3 of 67

in CIVIL LAW
Prepared by: Atty. Roney Jone P. Gandeza
pay the editor P50, 000 if the
editor will publish a false story
indicating that the politicians
opponent is gay. Suppose the
editor publishes the story, is he
entitled to collect the amount
promised?

oral agreement that is supports. The


writing need not be single document.
(A) is incorrect because the
Statute of Frauds requires only that
the party to be charged sign the
written agreement or note or
memorandum.
The
signature
requirement of the Statute of Frauds
is met if the writing contains the
signature of the party to be charged
or that of his agent. Therefore, both
parties do not need to sign the
writing to be enforceable under the
Statute.

No, because the agreement is


contrary to public policy. Considering
that the parties are in pari delicto,
the law will afford them no relief
against each other. The editor may
even be held criminally and civilly
liable for the libelous article.
11.

The Statute of Frauds:

(B) is incorrect because the


P500.00 threshold under the Statute
of Frauds is relevant only to
contracts for the sale of goods. (Art.
1403, 2(d), CC) Thus, a contract for
services is not within the Statute
even if the consideration is P500.00
or more, as long as it can be
performed in a year.

a. Mandates that both parties


sign the written contract
or note or memorandum
evidencing
their
agreement.
b. Applies to all contracts
under which payment of
P500.00 is to be made.

(D) is incorrect because the


Statute of Frauds contains no such
provisions. The Statute of Frauds
requires certain agreements to be
evidenced by a writing signed by the
party sought to be bound. Despite its
name, the Statute does not set forth
the elements of fraud as applicable
to contractual relationships.

c. Has no requirement that


the
agreement
of
the
parties be reduced to one
document.
d. Sets forth the elements of
fraud as applicable to
inducing another party to
enter into a contractual
relationship.

12. Why does the law impose an


obligation in the case of quasicontract?

(C) is correct. The Statute of


Frauds has no requirement that the
parties agreement be reduced to
one document. The Statute requires
the essential terms of certain
contracts to be evidenced by a
writing, a note or memorandum to
be enforceable. (Art. 1403, CC) Any
writing will suffice as long as it
contains every essential term of the

To prevent unjust enrichment.


In an action for unjust enrichment,
recovery
is based
upon
the
universally
recognized
moral
principle that one who has received
a benefit has the duty to make a
restitution when to retain such
benefit would be unjust.

I press toward
BAR OPERATIONS
2013 the mark for the

prize of the high calling of God


in Christ Jesus. - Philippians
3:14

BARRISTERS CLUB

Page 4 of 67

in CIVIL LAW
Prepared by: Atty. Roney Jone P. Gandeza
13. Pedro was seriously in a
vehicular accident. A bystander
called Dr. Garcia to render
medical treatment while Pedro
was unconscious. Dr. Garcia sent
Pedro a bill for the reasonable
value of his medical services.
Pedro refuses to pay. Judgment
for whom?

The
requirement
is
not
absolute. As a rule, the filing of an
action for legal separation requires a
cooling off period for six months.
(Art. 58, FC) However, when the
ground alleged os one of those
falling under RA 9262, also known as
the Anti-Violence Against Women
and their Children Act of 2004, there
is no such cooling off period
because the courts are mandated to
proceed with the hearing of the case
as soon as possible. (Sec. 19, Ra
9262)

Judgment for Dr. Garcia on the


basis of a quasi-contract. In the
problem presented, there was a
benefit bestowed to Pedro, a benefit
which is compensable under the law;
otherwise, Pedro will be unjustly
enriched at the expense of Dr.
Garcia.
14.
Abe took his car to a
carwash station and asked to
have it washed. While it is being
washed, Abe went to a nearby
mall for two hours. In the
meantime, one of the workers at
the car wash had mistakenly
hand-waxed the car. When Abe
came back, he was presented
with a bill for a wash job and a
hand wax. Is Abe liable to pay
for the hand wax job?

16.
A and B are Filipino
overseas workers in Syria. They
fell in love and decided to get
married in accordance with
Syrian law. On the day of the
wedding, A fell ill and could not
make it to the ceremony. Upon
advice of his Syrian lawyer, he
requested his best man to stand
as proxy during the wedding.
The marriage was celebrated in
accordance with Syrian law and
valid there as such. Is the
marriage valid here in the
Philippines?

No. the doctrine of quasicontract does not apply when there


is a contract between the parties.
Although there was a benefit
bestowed to Abe, Abe did not receive
an unjust benefit because the hand
wax job was not contemplated by his
agreement with the owner of the car
wash station.

Yes. Under Article 26 of the


Family Code, a marriage valid where
celebrated is valid in the Philippines.
To this general rule, the law
enumerates
exceptions,
but
a
marriage by proxy is not one of those
enumerated. Hence, the marriage is
valid. Expressio unius est exclusion
alterius.

15.
Article 58 of the Family
Code expressly provides that an
action for legal separation shall
in no case be tried before six
months shall have elapsed since
the filing of the petition. Is this
requirement absolute?

17.
H, a Filipino, marries W, an
American, in New York. At the
time of the marriage, H was
psychologically incapacitated to
enter into marriage, although
the incapacity manifested itself
five years after the wedding.

I press toward
BAR OPERATIONS
2013 the mark for the

prize of the high calling of God


in Christ Jesus. - Philippians
3:14

BARRISTERS CLUB

Page 5 of 67

in CIVIL LAW
Prepared by: Atty. Roney Jone P. Gandeza
Assuming that the marriage is
valid in New York, is it also valid
here?

estate which shall be taken from the


free portion.
Applying
the
foregoing
provisions, Anastacia is therefore
entitled to a legitime of P50, 000,
while Magdalena is entitled to a
legitime of P25, 000 which is to be
taken from the free portion. The
remainder of the estate constitutes
the free portion which goes to
Anastacia being the instituted heir.

No. The rule is:valid there,


valid here, except those prohibited
under Arts. 35 (1), (4), (5) and (6),
36, 37 and 38 of the Family Code.
The case falls squarely under one of
the exceptions enunciated in Article
26 of the Family Code; namely, a
void marriage under Article 36.
18.
Two
second
cousins,
Filipinos, got married in Japan
where such a marriage is void. Is
the marriage valid here?

20.
X and Y orally agreed that
X would lease office space to Y
at fixed rent as soon as the
current lessee of the premises
vacates in two years time.
Almost two years later, and
before he took possession of the
premises, Y learned of a much
more advantageous opportunity
and
established
his
office
elsewhere. To compel Y to
perform, X brought suit to
enforce the agreement.

No. The rule is:valid there,


valid here. Hence, void there, void
here also. This notwithstanding the
fact that had the marriage been
solemnized here, it would have been
perfectly valid.
19.
Abe is the legitimate son
of the widow, Anastacia. While
single, Abe made out a will
naming his mother as sole heir
to an estate valued at P100, 000.
Later, Abe married Magdalena.

Ys strongest argument in
his suit would be:
a. Mistake.
b.
Impossibility
of
performance
c. Statute of Frauds.
d. Autonomy of Contracts.
The Statute of Frauds is Ys
strongest
argument.
To
be
enforceable, the Statute requires
certain agreements to be evidenced
by a writing signed by the party to
be charged, including agreements
that can be performed within one
year from its making. Because the
lease in question is to be performed
after one year, the Statute of Frauds
is Ys strongest argument.
21. Tiger, Phil, and Rory are coowners of a parcel of land. Tiger
sells his undivided share in the

If Abe died while married


to Magdalena without changing
his will, would his entire estate
of P100, 000 go to his mother,
Anastacia?
Article 889 of the Civil Code
expressly provides that legitimate
parents or ascendants consist of
one-half of the hereditary estates of
their children and descendants.
Pursuant to Articlr 893, if the testator
leaves no legitimate descendants,
but leaves legitimate ascendants,
the surviving spouse shall have a
right to one-fourth of the hereditary

I press toward
BAR OPERATIONS
2013 the mark for the

prize of the high calling of God


in Christ Jesus. - Philippians
3:14

BARRISTERS CLUB

Page 6 of 67

in CIVIL LAW
Prepared by: Atty. Roney Jone P. Gandeza
property to Bubba. Phil later
sells his undivided share to
Matt. Is bubba entitled to
redeem Phils share?
Yes, because the right of legal
redemption is not limited to the
original co-owners. What matters is
that the redemptioner (Bubba) was
already a co-owner at the time when
another co-owner (Phil) sells his
undivided share. In the instant case,
Bubba was already a co-owner when
Phil sold his share.
22.
Suppose Phil, instead of a
sale, donated his undivided
share to Matt, may Bubba or
Rory, or both redeem?
No, because the right of legal
redemption may be exercised only if
the share of a co-owner is alienated
to a third person by onerous title.
23.
Suppose Rory later sells
his share to Bubba, may Matt
redeem?
No, because the right of legal
redemption may be exercised only if
the share of a co-owner is alienated
to a third person by onerous title.
The rationale behind the right
of legal redemption among coowners is to reduce the number of
co-owners and avoid entry of
strangers into the co-ownership. If
the alienation is in favor of a coowner, the number of co-owners is
already reduced and no stranger has
entered the co-ownership.
QUESTIONS
24-25
are
based on the following fact
situation:
Kobe, Lebron and Dwyane
are co-owners of a parcel of
land. Without the knowledge of
his co-owners, Kobe sells the
entire community property to
Kevin.
24.
Is the sale valid?
A co-owner who sells the
whole community property will affect

only his own share, and the


transferee gets only what would
correspond to his grantor in the
partition of the thing owned in
common.
By virtue of the sale of the
entire community property by a coowner, the buyer thereby becomes a
co-owner of the property to the
extent of participation of the seller
co-owner since the sale produced
the effect of substituting the buyer in
the enjoyment thereof.
In the instant case, Kevin thus
acquired only the rights pertaining to
the seller, Kobe, which is only onethird undivided share of the coowned lot.
25.
Suppose Kobe sells the
entire property to Kevin for
P600, 000, what would be the
basis of the redemption price?
The redemption price should
be one-third of the actual purchase
price, (P200, 000) not of the
prevailing market value of the
property.
Thus, Article 1088 of the Civil
Code provides that should any of
the heirs sell his hereditary rights to
a stranger before the partition, any
or all of the co-heirs may be
subrogated to the rights of the
purchaser by reimbursing him of the
price of the sale, provided they do so
within the period of one month from
the time they were notified in writing
of the sale by the vendor.
26.
Testator T died with a will.
He left no compulsory heirs and,
consequently, was completely
free to dispose of his properties,
without regard to legitimes. One
of the clauses in his will
perpetually
prohibits
the
alienation or mortgage by his
named legatees or devisees of
the properties specified therein.
Is the clause valid?

I press toward
BAR OPERATIONS
2013 the mark for the

prize of the high calling of God


in Christ Jesus. - Philippians
3:14

BARRISTERS CLUB

Page 7 of 67

in CIVIL LAW
Prepared by: Atty. Roney Jone P. Gandeza
No. The perpetual prohibition
is valid for only 20 years. The
testamentary provision stipulated in
Ts
will
prohibiting
perpetual
alienation of mortgage of the
properties
mentioned
therein
violated Articles 867 and 870 of the
Civil Code. Paragraph 4, Article 1013
of the same code which specifically
allows
a
perpetual
trust
in
inapplicable. Article 1013 is among
the Civil Code provisions on intestate
succession, specifically on the State
inheriting from a decedent, in default
of persons entitled to succeed. Under
this article, the allowance for a
permanent trust, approved by a
court of law, covers property
inherited by the State by virtue of
intestate succession. The article
does not cure a void testamentary
provision which did not institute an
heir.
In the instant case, Ts estate
cannot be subjected indefinitely to a
trust because the ownership thereof
would then effectively remain with
him even in the afterlife. (Orendain
v. Estate of Rodriguez, June 30,
2009)

diligence in the selection and


supervision of his employees to
avoid liability?

QUESTIONS
27-30
based on the following
situation:

30.
May
damages
owners?

No, he cannot. This is so


because his liability as a common
carrier is based on breach of
contract. Such a defense will only
serve to mitigate As liability
because by then he will be
considered as a debtor in good faith.
28.
May B, owner of the cargo
truck, invoke the same defense?
Yes, because his liability is
based on a quasi-delict.
29.
May
damages
owners?

the bus
invoke

claim
both

moral
vehicle

C can claim moral damages


against B, owner of the cargo truck,
because of the injuries he suffered,
but as against A, owner of the bus, C
can claim moral damages only if he
proves reckless negligence on the
part
of
the
common
carrier
amounting to fraud.

are
fact

A passenger bus owned by


A and a cargo truck owned by B
collided. C, a bus passenger,
suffered
injuries,
while
D,
another bus passenger, died.
The drivers of the two vehicles
were at fault. C, the injured bus
passenger, and E, the surviving
heir of D, immediately filed suit
against the owners of both
vehicles for damages.
27.
May A,
successfully

C
from

E
from

claim
both

moral
vehicle

E can claim moral damages


against both vehicle owners because
the rules on damages arising from
death due to a quasi-delict are also
applicable to death of a passenger
caused by breach of contract by a
common carrier. (Arts. 1755, 1764,
2206 and 2219, CC)
31. To satisfy a final judgment
rendered against H and W,
husband and wife, the sheriff
levied on a house and lot
constituted by the couple as

owner,
due

I press toward
BAR OPERATIONS
2013 the mark for the

prize of the high calling of God


in Christ Jesus. - Philippians
3:14

BARRISTERS CLUB

Page 8 of 67

in CIVIL LAW
Prepared by: Atty. Roney Jone P. Gandeza
their family home. The sheriff
later sold the property at public
auction to C, the judgment
creditor.

32.
What is an easement of
aqueduct? Does the existence of
an easement of right of way
necessarily
include
the
easement of aqueduct?

After some discussion, C


agreed to allow the couple to
remain on the property as
lessees and to vacate the same
in two years. A year later, the
couple brought an action to
annul the auction sale on the
allegation that the property is
their family home and its sale is
void ab initio.

An easement of aqueduct is
the right to make water flow thru
intervening estates in order that one
may make use of said waters.
The existence of the easement
of right of way does not necessarily
include the easement of aqueduct.
Consequently, the right to dig
trenches and to lay pipelines for the
conducting of water is not included
in a contract granting a right of way
(the rights given merely those of
ingress and egress to and from the
lot involved).

Will the action prosper?


The action will not prosper.
Even if the subject property is a
family home and, thus, should have
been exempt from execution, H and
W should have asserted it as a family
home and its being exempt from
execution at the time it was levied
upon or within reasonable time
thereafter. Failure to do so would
estop the spouses from later
claiming the exemption.

33.
X promised to donate his
friend, Y, a parcel of land.
Relying on such promise, Y
constructed a house of strong
materials on the land. When X
died,
however,
his
son
Z
inherited the land. In the suit
filed by Z to recover possession
of the land from Y, the latter
invoked
the
right
to
be
reimbursed of his necessary and
useful improvements on the
allegation that he is a builder in
good faith.

It is evident that H and W did


not assert their claim of exemption
within a reasonable time. At no other
time can the status of a residential
house as a family home be set up
and proved and its exemption from
execution claimed but before the
sale thereof at public auction.

Is Y a builder in good faith?

Settled is the rule that the


right of exemption is a personal
privilege granted to the judgment
debtor and as such, it must be
claimed not by the sheriff but by the
debtor himself at the time of the levy
or
within
a
reasonable
time
thereafter. (De Mesa v. Acero, G.R.
No. 185064, January 16, 2012)

No. The mere promise of X to


donate the property to Y does not
make the latter a builder in good
faith. This is because at the time
when the improvement was built,
such promise was not yet fulfilled. It
was therefore a mere possession by
tolerance.

I press toward
BAR OPERATIONS
2013 the mark for the

prize of the high calling of God


in Christ Jesus. - Philippians
3:14

BARRISTERS CLUB

Page 9 of 67

in CIVIL LAW
Prepared by: Atty. Roney Jone P. Gandeza
A person whose occupation of
a realty by sheer tolerance of its
owner is not a possessor in good
faith; hence, not entitled to the value
of the improvements built thereon.
(Verno Padua-Hilario v. Court of
Appeals, et.al., January 19, 2000)
34.
S contracts to sell to B a
parcel of land. They agree that B
shall pay the purchase price on
October 25, and that in case of
Bs failure to pay, the contract
shall be automatically rescinded.
If B does not pay on October 25,
can he still pay on October 29?
Yes, provided there has been
no judicial or notarial demand for
rescission of the contract as of
October 29. Under Article 1592 of
the Civil Code, in the sale of an
immovable property, even though it
may have been stipulated that upon
failure to pay the price at the time
agreed upon the rescission of the
contract shall of right take place, the
vendee
may
pay
even
after
expiration of the period, as long as
no demand for rescission of the
contract has been made upon him
either judicially or by a notarial act.
35.
In a contract to sell, is it
necessary for the vendor to send
a notarial rescission when the
vendee fails to pay the balance
of the purchase price?
No.
Rescission,
whether
judicially or by notarial act, is not
required to be done by the vendor.
There can be no rescission of an
obligation that is still non-existing,
the suspensive condition not having
happened.
In a contract to sell, there is
no contract to rescind, judicially or

by a notarial act, because from the


moment the vendee fails to pay on
time the purchase price, the contract
between the parties is deemed ipso
facto rescinded.
36.
In a contract of sell, is the
vendor obliged to refund what
the vendee paid under the
contract if the sale is not
consummated?
Yes. The partial payment
made by the buyer must be returned
to him, in the absence of a
stipulation regarding forfeiture of
payments made by the parties. Such
action is but just and equitable under
the premises. If it were otherwise,
there will be unjust enrichment on
the part of the seller at the expenses
of the buyer. (Ordenv. Aurea, G.R.
No. 172733 August 20, 2008)
37.
When a co-owner seeks to
exercise his right of legal
redemption, within what period
may he exercise such right?
The right of legal redemption
(or pre-emption) shall be exercised
by a co-owner within 30 days from
written notice by the seller-co-owner.
Actual knowledge notwithstanding,
written notice is still required.
EXCEPTION: Actual knowledge by
co-heirs or co-owners living in the
same land with purchaser; or coowner was middleman in sale to
third party, the requirement of
written notice is dispensed with;
period of redemption begins to run
from actual knowledge.
QUESTIONS
37-39
based on the following
situation:

I press toward
BAR OPERATIONS
2013 the mark for the

prize of the high calling of God


in Christ Jesus. - Philippians
3:14

BARRISTERS CLUB

Page 10 of 67

are
fact

in CIVIL LAW
Prepared by: Atty. Roney Jone P. Gandeza
A sold an unregistered
land to B. Upon As death, his
brother
C,
executed
an
extrajudicial settlement of As
estate under which he (C)
adjudicated
exclusively
unto
himself
the
lot
and
simultaneously sold the same to
D who was unaware of the prior
sale.
37.
Is the rule on double sale
under Article 1544 of the Civil
Code applicable to the above
problem?
No. Article 1544 if the Civil
Code on double sales applies only
where the same thing is sold to
different vendees by the same
vendor. It does not apply where the
same thing is sold to different
vendees by different vendors as in
the case at bar.
38.
Who between the two
buyers is the rightful owner of
the lot?
B is the rightful owner of the
lot. When A sold to B the property,
ownership thereof was transferred to
B in accordance with Article 1496 of
the Civil Code which provides that
the ownership of the thing sold is
acquired by the vendee from the
moment it is delivered to him in any
of the ways specified in Articles 1497
to 1501.
Article 1498, in turn, provides
that when the sale is made through a
public instrument, the execution
thereof shall be equivalent to the
delivery of the thing which is the
object of the contract, if from the
deed the contrary does not appear
or cannot be clearly inferred. In the

problem presented, the sale between


A and B contains nothing contrary to
intent to transfer ownership.
When A died in 1993, she no
longer owned the lot, and, therefore,
his brother could not have inherited
it. The Extrajudicial Settlement with
Simultaneous Sale did not thus
confer upon C ownership of the lot in
question; hence he could not have
conveyed it to D.
39.
Is the issue of Ds good
faith relevant in solving whether
he has a preferential right to the
lot?
The issue of good faith or bad
faith is relevant only where the
subject of the sale is a registered
land and the purchaser is buying the
same from the registered owner
whose title to the land is clean. In
such case the purchaser who relies
on the clean title of the registered
owner is protected if he is a
purchaser in good faith and fro
value. (Ong v. Olasiman, 465 SCRA
464 [2006])
40.
Article 40 of the Family
Code specifically provides that
the absolute nullity of a previous
marriage may be invoked for
purposes of remarriage on the
basis solely of a final judgment
declaring
such
previous
marriage void. What are the
consequences of this provision?
a) Parties to a marriage are
not permitted to judge for
themselves the nullity of
their marriage. Only the
competent courts have
that authority.
b) Prior to such declaration,
the validity of the first

I press toward
BAR OPERATIONS
2013 the mark for the

prize of the high calling of God


in Christ Jesus. - Philippians
3:14

BARRISTERS CLUB

Page 11 of 67

in CIVIL LAW
Prepared by: Atty. Roney Jone P. Gandeza
marriage
question.

is

beyond

c) A party who contracts a


second marriage assumes
the
risk
of
being
prosecuted for bigamy.
d) If the first marriage is void
but a party remarries
without seeking judicial
nullity of his first marriage,
the second marriage is also
void for non-compliance
with Article 40 in relation
to Articles 51 to 53 of the
Family Code.
e) Bigamy is committed as
long as the first marriage,
although has not been
declared as such under
Article 40. (Mercado v. Tan,
337 SCRA 122)
f) Bigamy is committed even
if the second marriage is
declared
void
on
the
ground of psychological
incapacity of one of the
spouses
because
such
second marriage, although
void, still produces legal
consequences,
among
which is incurring criminal
liability
for
bigamy.
(Tenebro
v.
Court
of
Appeals, 424 SCRA 272)
g) If the second marriage took
place before the Family
Code, Article 40 does not
apply. Consequently, there
is no need for a judicial
decree of nullity of the first
marriage. (Ty v. Court of
Appeals, 346 SCRA 86)

h) If the second marriage took


place after the Family
Code, even if the first
marriage took place before
it,
Article
40
applies
because it has retroactive
effect.
(Atienza
v.
J.
Brillantes, A.m. No. MTJ-92706, 243 SCRA 32)
41. S orally offers to sell his land
to B for 300, 000. B accepts the
offer and pays s the purchase
price. S in turn delivers to B the
title of the land. B now seeks to
register the land in his name but
the Register of Deeds refuses to
register the sale in the absence
of a notarized deed of sale.
Which of the
accurately describes
remedy?

a) He may enter into the


possession of the land
as a buyer in good faith.
b) He cannot compel S to
return
the
payment
because the contract is
unenforceable.
c) He may compel S to
execute
a
notarized
deed of sale because
the contract is valid and
enforceable.
d) He may sue S to return
the
purchase
price
because no one may
enrich himself at the
expense of another.
(C) is Bs best available
remedy. The notarized deed of sale is
required demanded by B is neither
for validity nor for enforceability of

I press toward
BAR OPERATIONS
2013 the mark for the

prize of the high calling of God


in Christ Jesus. - Philippians
3:14

BARRISTERS CLUB

following
Bs best

Page 12 of 67

in CIVIL LAW
Prepared by: Atty. Roney Jone P. Gandeza
the contract of sale. The sale is both
valid and enforceable. The subject
document is required by B only for
his convenience; i.e., to allow him to
register the sale.
42.
Article 213 of the Family
Code enunciates the rule that no
child below the age of seven
years shall be separated from
the
mother,
except
for
compelling reasons. In custody
cases,
when
should
this
provision be applied by the
courts at the time of the filing
of the petition for custody or at
the time when the court is to
decide who between the parents
is entitled to the custody of the
child?
The argument that the 7-year
reference in the law applied to the
date when the custody case is filed,
not the date when the decision is
rendered, is flawed. The matter of
custody
is
permanent
and
unalterable. If the parent who was
given custody (either by law or by
choice of the child) suffers a future
character change and becomes unfit,
the matter of custody can always be
re-examined and adjusted. (Espiritu,
et.al., v. Court of Appeals., 242 SCRA
362 [1995])
QUESTIONS
43-44
based on the following
situation:

are
fact

X executes a real estate


mortgage on his land in favor of
Y to answer for any deficiency
that may result from foreclosure
of
the
chattel
mortgage
constituted over the car sold on
installments by Y to Z.

43.
Assuming
there
is
a
deficiency after foreclosure of
the chattel mortgage, may Y
foreclose
the
real
estate
mortgage constituted on Xs
land?
No, because in such a case, the
third person-mortgagor (X), after
paying
the
deficiency
through
foreclosure
of
the
real
estate
mortgage, has the tight of a
guarantor who can hold the vendee
(z) liable for the payment made, thus
indirectly violating the prohibition
under the law. (Art, 1484 (3), Civil
Code)
44.
Suppose in the preceding
problem Y assigns the promissory
note to M, promising the latter
that should Z default and the
chattel mortgaged is foreclosed
resulting in a deficiency, the
assignor (Y) shall answer for the
deficiency to the assignee (M). Is
this stipulation valid?
Yes, because in such a case, it
is no longer the vendee (Z) who is
held liable but the vendor (Y). There
is this no violation of the Recto Law
that if the vendor avails himself of the
right to foreclose, he is prohibited
from bringing an action against the
purchase for the unpaid balance.
45.
What are the instances
under the law when form is an
indispensable
and
mandatory
requirement for the validity of
the contract?
Form
is
a
mandatory
requirement for the validity of the
following contracts:
(1) If the value of the personal
property donated exceeds

I press toward
BAR OPERATIONS
2013 the mark for the

prize of the high calling of God


in Christ Jesus. - Philippians
3:14

BARRISTERS CLUB

Page 13 of 67

in CIVIL LAW
Prepared by: Atty. Roney Jone P. Gandeza
P5, 000, the donation and
the acceptance shall be
made in writing: otherwise,
the donation is void. (par. 3,
Art. 748, CC);
(2) Donation of an immovable,
regardless of value, must be
in a public instrument (Art
748, CC);
(3) A contract of partnership is
void whenever property is
contributed thereto, if an
inventory of said property is
not made, signed by the
parties, and attached to the
public
instruments
(Art.
1773, CC);
(4) Sale of piece of
through an agent
1847, CC);

land
(Art.

(5) Antichresis (Art. 134, CC);


and
(6) Payment of interest (Art.
1956, CC).
46.
If a marriage is declared
void, how should the properties
acquired by the spouses during
the marriage be liquidated,
partitioned, and distributed?
If a marriage is declared void,
the properties of the spouses should
be owned in the concept of coownership. A void or voidable
marriage, regardless of its ground,
cannot be governed by the conjugal
partnership of gains nor by the
absolute community of property
regime. In such a case, Article 147of
the Family Code applies, except when
the marriage is declared void for
being bigamous, in which case Article
148 governs.

(RJPG:
The above rules
assumes significance in light of
Articles 102 (absolute community or
property)
and
129
(conjugal
partnership) of the Family Code which
commonly provide that the conjugal
dwelling shall be adjudicated to the
spouse with whom the majority of the
common children choose to remain.
Applying the above rule, if a marriage
is declared void, the conjugal dwelling
shall be partitioned not in accordance
with Articles 102 and 129, but in
accordance with Articles 147 and 148
of the Family Code. This holds true
even if majority of the children
choose to remain with one parent.)
47.
In an action for annulment
of marriage, who are the parties
who ay commence the action and
within what period may the
action be filed?
The following are the parties
who may commence an action for
annulment of marriage and the
periods for the filing of such action:
a) LACK OF PARENTAL CONSENT: The
minor should bring the action within
five years after attaining the age of
21. For the parent or guardian, the
action must be brought at any time
before such party reaches the age of
21.
b) INSANITY: The sane spouse or
person having legal charge of the
insane spouse must bring the action
at any time before the death of either
party. The insane spouse must bring
the action during a lucid interval or
after regaining sanity, also before the
death of the other party. The reason
in not providing for a 5-year period is
the insanity recurs.

I press toward
BAR OPERATIONS
2013 the mark for the

prize of the high calling of God


in Christ Jesus. - Philippians
3:14

BARRISTERS CLUB

Page 14 of 67

in CIVIL LAW
Prepared by: Atty. Roney Jone P. Gandeza
c) FRAUD: Injured party must bring the
action
within
five
years
after
discovery of the fraud.
d) FORCE, INTIMIDATION, UNDUE
INFLUENCE: The injured party must
bring the action within five years from
the time the force, intimidation or
undue influence disappeared or
ceased.
e) PHYSICAL INCAPACITY: The injured
party must bring the action within
five years after the marriage.
f) SEXUALLY-TRANSMISSIBLE
DISEASE: The injured party must
bring the action within five years after
the marriage.
48.
Under
the
Domestic
Adoption Act, may the adopting
parent file for rescission of the
decree of adoption?

wedding. A month following


Debbys birth, Cora died in a car
accident. Andy reunited with
Betty and immediately married
her. Is Debby legitimated by the
marriage of her parents?
Yes. It is clear from the problem
that although Debby was conceived
and born outside of wedlock of her
parents, she was conceived at the
time when her parents, Andy and
Betty, were not disqualified by any
impediment to marry each other. The
reckoning point for legitimation is the
period of conception the child, not his
birth, and the subsequent valid
marriage of the parents.
49.
What
is
the
new
interpretation
given
to
the
second paragraph of Article 26 of
the Family Code?

No. The Domestic Adoption Act


had already abrogated the right of
the adopting parent to rescind a
decree of adoption. Nevertheless, the
adopting parent can always, for valid
reasons, cause forfeiture of certain
benefits that would otherwise accrue
to the adopted. For instance, upon
the grounds provided by law, an
adopter may deny the adopted his
legitime and by a will, may exclude
him from having a share in the
disposable free portion. (Lahom v.
Sibulo, GR No. 143989, July 14, 2003)

In Republic v. Orbecido III, 472


SCRA 114 [2005], the Supreme Court
made a novel pronouncement that
paragraph 2 of Article 26 of the
Family Code should be interpreted to
include cases involving parties who,
at the time of the marriage, were
Filipino citizens but later one of them
becomes a naturalized citizen of a
foreign country and obtains a divorce
decree. The Filipino spouse should
likewise be allowed to remarry as if
the other party were a foreigner at
the time of the marriage. To rule
otherwise will be a sanction absurdity
and injustice.

49.
Andy
and
Betty,
both
eligible to marry each other,
cohabited as husband and wife
without the benefit of marriage.
A few weeks before Betty was to
give
birth,
however,
Andy
married Cora, an old maid.
Heartbroken, Betty gave birth to
Debby a few weeks after Andys

ANOTHER ANSWER: As pointed


out by the Supreme Court in
Obrecido, the reckoning point is not
the citizenship of the parties at the
time of the celebration of the
marriage, but their citizenship at the
time a valid divorce is obtained

I press toward
BAR OPERATIONS
2013 the mark for the

prize of the high calling of God


in Christ Jesus. - Philippians
3:14

BARRISTERS CLUB

Page 15 of 67

in CIVIL LAW
Prepared by: Atty. Roney Jone P. Gandeza
abroad by alien spouse capacitating
the latter to remarry.
QUESTION 50-51 are based
on the following fact situation:
William, an American, and
Marissa, a Filipina, cohabited as
husband and wife without the
benefit of marriage. During their
cohabitation, the couple bought
from Mauricio a parcel of land in
Baguio City. Although the deed of
sale was placed in the names of
both William and Marissa as
buyers, the sale was registered
in the name of Marissa alone
because William was disqualified
to own real properties in the
Philippines.
It
is
sufficiently
established that the funds used
to buy the property came solely
from William, as Marissa has no
sufficient source of income.
After their relationship has
turned sour and the two went
separate ways, William sold all
his rights and interests in the
property to Nicasio, a Filipino.
When
Nicasio
tried
to
register the property in his name,
he discovered that the certificate
of title is already registered in
the name of Marissa, and that it
has already been mortgaged.
50. If William is the true buyer of
the property, what is the effect of
the registration of the property
in the name of Marissa?
The registration of the property
in the name of Marissa does not make
her the owner of the property in
question. It is settled that registration
is not a mode of acquiring ownership.
It only means of confirming the fact of

its existence with notice to the whole


world at large. Certificates of title are
not a source of right. The mere
possession of a certificate of title
does not make the holder the true
owner of the property. Thus, the mere
fact that Marissa has the title of the
disputed property in her name does
not necessarily, conclusively, and
absolutely make her the owner.
51.
Given
that
William
is
disqualified
to
own
real
properties in the Philippines, is
the sale between him and Nicasio
valid?
Given
that
aliens
are
disqualified to own real properties in
the Philippines, the transfer of the
subject property from Mauricio to
William, who is an American, would
have
been
declared
invalid if
challenged, had not William conveyed
the property to Nicasio who is a
Filipino citizen. It has been the
consistent ruling of the Supreme
Court that if the land is invalidly
transferred
to
an
alien,
who
subsequently becomes a Filipino
citizen, or transfers it to a Filipino, the
flaw in the original transaction is
considered cured and the title of the
transferee
is
rendered
valid.
(Borromeo v. Descallar, GR No.
159310, Febraury 24, 2009)
52.
John, an American, and
Nieves, a Filipina, were married
in Cebu City in 1997. During their
marriage, Nieves bought from
Dionisia a beach lot in Boracay
for P1.2 million. The sale was
financed by John. The couple,
also
using
Johns
funds,
introduced improvement on the
beach
lot
and
eventually
converted the property into a
vacation and tourist resort.

I press toward
BAR OPERATIONS
2013 the mark for the

prize of the high calling of God


in Christ Jesus. - Philippians
3:14

BARRISTERS CLUB

Page 16 of 67

in CIVIL LAW
Prepared by: Atty. Roney Jone P. Gandeza
When the couple had a
failing out, Nieves leased the
property to Matthews for a
period of 25 years. She did so
without Johns consent.
Does John have the legal
standing to question the validity
of the lease agreement on the
theory that in so doing, he was
merely exercising his prerogative
as a husband regarding conjugal
property?
John, being an alien, is
absolutely prohibited from acquiring
public and private lands in the
Philippines. Considering that Nieves
appeared to be the designated
vendee of the property leased, she
acquired sole ownership thereto. This
is true even of Johns claim that he
provided
the
funds
for
such
acquisition is to be sustained. By
entering into such contract knowing
that it was illegal, no implied trust
was created in his favor; no
reimbursement for his expenses can
be allowed; and no declaration can be
made that the subject property was
part of the conjugal or community
property of the spouses. In any event,
he had and has no capacity or
personality
to
question
the
subsequent lease of the property by
his wife in the theory that in so doing,
he was merely exercising the
prerogative of a husband in respect of
conjugal property. To sustain such a
theory would countenance indirect
violation
of
the
constitutional
prohibition. If the property were to be
declared conjugal, as he would then
have a decisive vote as to its transfer
or disposition. This is a right that the
Constitution does not permit him to
have. Thus, the validity of the lease

must be upheld. (Mathews v. Taylor,


June 22, 2009)
53.
What are the recognized
exceptions
under
the
Constitution allowing aliens to
own
private
lands
in
the
Philippines?
The instances when aliens are
allowed to acquire private lands in
the Philippines are:
(a) By hereditary succession (Section 7,
Article XII, 1987 Constitution);
(b) A
natural-born
citizen
of
the
Philippines who has lost his Philippine
citizenship may become a transferee
of private lands, subject to the
limitations provided by law. (Section
8, Article XII, Constitution). (Republic
Act No. 8179 now allows a former
natural-born Filipino citizen to acquire
up to 5,000 square meter of urban
land and 3 hectares of rural land, and
he may now use the land not only for
residential purposes, but even for
business or other purpose); and
(c) Americans who may have acquired
title to private lands during the
effectivity of the Parity Agreement
shall hold valid title thereto as against
private persons (Section 11, Article
XII, 1973 Constitution).
54.
Mao Tse Tung, a Chinese
national, bought from Juan dela
Cruz, a Filipino, a parcel of land
in Bulacan. Upon Maos death,
the property was transferred by
succession to his son, Chiang Kai
Shek. When Chiang died, the
property was in turn transferred
by succession to his Filipino wife,
Maria.
May
successfully

the
government
prosecute
a

I press toward
BAR OPERATIONS
2013 the mark for the

prize of the high calling of God


in Christ Jesus. - Philippians
3:14

BARRISTERS CLUB

Page 17 of 67

in CIVIL LAW
Prepared by: Atty. Roney Jone P. Gandeza
reversion case on the argument
that the sale of the land to Mao
violated
the
Constitutional
provision disallowing aliens from
acquiring public and private
lands in the Philippines?
If
the
government
had
commenced reversion proceedings
when the lot was still in the hands of
Mao Tse Tung who was an alien
disqualified to hold title thereto, the
reversion of the land to the State
would
undoubtedly
be
allowed.
However, this is not the case here.
When the government instituted the
reversion case, the lot had already
been transferred by succession to
Maria who is a Filipino citizen. And
since the lot was transferred to a
Filipino citizen, the flaw in the original
transaction is considered cured.
(Republic v. Register of Deeds, GR
158230, July 16, 2008)

brother Nicanor and their father


Vicente.
Is
this
argument
tenable?
The contention that an implied
trust was created between Nicanor
and his father does not hold water
because the prohibition against an
alien from owning lands of the public
domain is absolute and not even an
implied trust can be permitted to
arise on equity considerations.
In the instant case, Nicanor
became the owner of the subject
when he was granted a miscellaneous
sales patent by the Bureau of Lands.
Under the law, a certificate of title
issued pursuant to any grant or
patent involving public land is a
conclusive and indefeasible as any
other certificate of title issued to
private lands in the ordinary or
cadastral registration proceeding.

55.
Vicente Ting, a Chinese
national married to a Filipina,
occupied
a
public
land
in
Olongapo City. Pursuant to an
affidavit,
Vicente
transferred,
without valuable consideration,
all his rights and interests over
the lot in favor of his eldest son,
Nicanor. On the basis of the
affidavit, Nicanor, who earlier
obtained Filipino citizenship, was
issued a miscellaneous sales
patent by the Bureau of Lands.
Not long after, the Register of
Deeds
issued
an
original
certificate of title over the
property in the name of Nicanor.

The effect of the registration of


a patent and the issuance of a
certificate of title to the patentee is to
vest in him an incontestable title to
the land, in the same manner as if
ownership had been determined by
final decree of the court, and the title
so issued is absolutely conclusive and
indisputable, and is not subject to
collateral attack, consequently, the
lot is not part of Vicente Tings estate.
(Ting Ho, Jr. v. Teng Gui, GR No.
130115, July 16, 2008)

Upon
Vicentes
death,
Nicanors brothers and sisters
filed for judicial partition of the
lot on the allegation that an
implied trust over said property
was
created
between
their

Marcel and Monique, French


nationals, entered into a Contract
to Sell with Fil-Estate Realty
Corp. for the purchase of a
residential unit in a townhouse
project. When Fil-Estate failed to

QUESTIONS
56-57
based on the following
situation:

I press toward
BAR OPERATIONS
2013 the mark for the

prize of the high calling of God


in Christ Jesus. - Philippians
3:14

BARRISTERS CLUB

Page 18 of 67

are
fact

in CIVIL LAW
Prepared by: Atty. Roney Jone P. Gandeza
comply with its verbal promise to
complete the project at a certain
date,
the
couple
filed
for
rescission of the contract.
56.
What it the status of the
contract entered into by Marcel
and Monique with Fil-Estate
Realty Corp.?
The Contract to Sell is void.
Since Marcel and Monique, being
French nationals, are prescribed
under the Constitution from acquiring
and owning real property, it follows
that the Contract to Sell entered into
by the parties is void. Under the Civil
Code, all contracts whose cause,
object or purpose is contrary to law or
public policy and those expressly
prohibited or declared void by law are
inexistent
and
void
from
the
beginning.
A
void
contract
is
equivalent to nothing; it produces no
civil effect.
57.
In any event, are Marcel
and Monique entitled to recover
from Fil-Estate the amount paid
as well as interest and other
damages?
Since the contract involved
here is a Contract to Sell, ownership
has not yet transferred to Marcel and
Monique when they filed the suit for
rescission. While the intent to
circumvent
the
constitutional
prescription on aliens owning real
property was evident by virtue of the
execution of the Contract to Sell, such
violation if the law did not materialize
because the spouses caused the
rescission of the contract before the
execution
of
the
final
deed
transferring ownership.
Thus, the exception to the
application of the pari delicto doctrine

finds application in the case at bar.


Under the law, one who repudiates
the agreement and demands his
money before the illegal act has
taken place is entitled to recover.
Marcel and Monique are therefore
entitled to the recovery only of the
purchase price. No damages may be
recovered on the basis of a void
contract; being nonexistent, the
agreement produces no juridical tie
between the parties involved. Further,
the spouses are not entitled to actual
as well as interest thereon, moral and
exemplary damages and attorney
fees. (Hulst v. PR Builders, Inc., 532
SCRA 41 [2007])
58.
H and W were married in
1978. In 1992, H obtained a
decree of legal separation after
catching his wife having illicit
relations with their neighbor. In
the
final
decree
of
legal
separation issued by the court,
the court ordered the forfeiture
of Ws share in the net profits
earned
by
the
conjugal
partnership in favor of her
children pursuant to Article 63(2)
in relation to Article 129 of the
Family Code.
Not satisfied with the ruling
regarding the forfeiture of her
share in the conjugal assets, W
claims that the net assets of the
conjugal partnership shall be
computed in accordance with
Article 102 of the Family Code (a
provision
under
absolute
community of property), instead
of Article 129 (a provision on
conjugal
partnership).
She
argues that Article 102 applies
because there are no other
provisions under the Family Code
which defines net profits subject
of forfeiture as a result of legal

I press toward
BAR OPERATIONS
2013 the mark for the

prize of the high calling of God


in Christ Jesus. - Philippians
3:14

BARRISTERS CLUB

Page 19 of 67

in CIVIL LAW
Prepared by: Atty. Roney Jone P. Gandeza
separation. She contends that
her vested right over half of the
common
properties
of
the
conjugal partnership is violated
when the forfeiture is to be made
pursuant
to
Article
129
in
relation to Article 63(2) of the
Family Code.
Ws
move
following issues:

raises

the

a) What
law
governs
the
property relations of the
spouses given that they
were married before the
effectivity of the Family
Code?
b) What
law
governs
the
dissolution of their common
properties since the decree
of legal separation was
issued after the Family
Code is already in effect?
As to their property relations,
the
Spouses are governed by the regime
of conjugal partnership of gains. This
is so because they were married
when the operative law was the Civil
Code. As to the liquidation of their
conjugal partnership assets, however,
the Family Code is applicable because
it is already the operative law at the
time of the dissolution of their
conjugal partnership.
In the instant case, the
applicable law in so far as the
liquidation of the conjugal partnership
assets and liabilities of H and W is
concerned is Article 129 of the Family
Code (liquidation of the conjugal
partnership) in relation to Article 63
(effects of a decree of legal
separation). The latter provision is
applicable because insofar as Article

256 of the Family Code [t]his code


shall have retroactive effect insofar as
it does not prejudice or impair vested
or acquired rights in accordance with
the Civil Code or other law.
Ws contention that her vested
right over half of the common
properties of the conjugal partnership
is violated when her share in the
conjugal partnership is forfeited in
favor of her children pursuant to
Article 63(2) and 129 of the Family
Code has no basis.
While it is true that the couple
were married at the time when the
operative law was the Civil Code, the
Family
Code
should
be
given
retroactive application for purposes of
determining the net profits earned
by the conjugal partnership which is
subject to forfeiture. A spouses claim
of a vested right is not etched in
stone. To be vested, a right must
have become a title legal or
equitable to the present or future
enjoyment of property. In one case,
the Supreme Court reiterated its long
standing ruling that prior to the
liquidation
of
the
conjugal
partnership, the interest of each
spouse in the conjugal assets is
inchoate, a mere expectancy, which
constitutes neither a legal nor an
equitable estate, and does not ripen
into title until it appears that there
are assets in the community as a
result
of
the
liquidation
and
settlement. The interest of each
spouse is limited to the net remainder
resulting from the liquidation of the
affairs of the partnership after its
dissolution. Thus, the right of the
husband or wife to one-half of the
conjugal assets does not vest until
the dissolution of the marriage, when
it is finally determined that, after
settlement of conjugal obligations,

I press toward
BAR OPERATIONS
2013 the mark for the

prize of the high calling of God


in Christ Jesus. - Philippians
3:14

BARRISTERS CLUB

Page 20 of 67

in CIVIL LAW
Prepared by: Atty. Roney Jone P. Gandeza
there are net assets left which can be
divided between the spouses or their
respective heirs.
59.
Is the computation of net
profits earned in the conjugal
partnership of gains the same as
the computation of net profits
earned
in
the
absolute
community?
The term net profits is defined
in Article 102(4) of the Family Code.
Under this provision, the term net
profits shall be the increase in value
between the market value of the
community property at the time of
the celebration of the marriage and
the market value at the time of its
dissolution. Without any doubt,
Article 102(4) applies to both the
dissolution of the absolute community
regime under Article 102 of the
Family Code, and to the dissolution of
the conjugal partnership regime
under Article 129 of the Family Code.
The difference lies in the process
used under the dissolution of the
absolute community regime under
Article 102 of the Family Code, and in
the processes used under the
dissolution
of
the
conjugal
partnership regime under Article 129
of the Family Code.
ON ABSOLUTE COMMUNITY
REGIME: Applying Article 102 of the
Family Code, the net profits requires
a prior determination of the market
value of the properties at the time of
the communitys dissolution. From
the totality of the market value of all
the properties, the debts and
obligations
of
the
absolute
community are to be deducted and
this will result to the net assets or net
remainder of the properties of the
absolute community, from which the
value of the properties at the time of

marriage is to be deducted, which


then results to the net profits.
ON
CONJUGAL
PARTNERSHIP REGIME: Applying
Article 129 of the Family Code, the
net
profits
requires
a
prior
determination
of
the
separate
properties and debts of the spouses
under the following procedure a) an
inventory shall be prepared, listing
separately all the properties of the
conjugal
partnership
and
the
exclusive properties of each spouse;
b) amounts advanced by the conjugal
partnership in payments of personal
debts and obligations of either spouse
shall be credited to the conjugal
partnership as an asset thereof; c)
each spouse shall be reimbursed for
the use of his or her exclusive funds
in the acquisition of property or for
the value of his or her exclusive
property, the ownership of which has
been vested by law in the conjugal
partnership; d) the debts and
obligations
of
the
conjugal
partnership shall be paid out of the
conjugal
assets.
In
case
of
insufficiency of said assets, the
spouses shall be solidarily liable for
the unpaid balance with their
separate properties, in accordance
with the provisions of paragraph 2 of
Article 121; e) whatever remains of
the exclusive properties of the
spouses shall thereafter be delivered
to each of them; f) unless the owner
had been indemnified from whatever
source, the loss or deterioration of
movables used for the benefit of the
family, belonging to either spouse,
even due to fortuitous event, shall be
paid to said spouse from the conjugal
funds, if any; and g) the net
remainder of the conjugal partnership
shall constitute the profits, which
shall be divided equally between
husband and wife, unless a different

I press toward
BAR OPERATIONS
2013 the mark for the

prize of the high calling of God


in Christ Jesus. - Philippians
3:14

BARRISTERS CLUB

Page 21 of 67

in CIVIL LAW
Prepared by: Atty. Roney Jone P. Gandeza
proportion or division was agreed
upon in the marriage settlements or
unless there has been a voluntary
waiver or forfeiture of such share as
provided in the Family Code.
60.
Suppose Article 102 of the
Family Code (which is a provision
under the regime of absolute
community of property) is to
apply in the instant case, is W
entitled to receive anything from
the absolute community?
If H and W have no separate
properties, the remaining properties
of the couple are all part of the
absolute community. And its market
value at the time of the dissolution of
the absolute community constitutes
the market value at dissolution.
When H and W were legally
separated, all the properties which
remained will be liable for the debts
and obligations of the community.
Such debts and obligations will be
subtracted from the market value at
dissolution. What remains after the
debts and obligations have been paid
from the total assets of the absolute
community
constitutes
the
net
remainder or net asset. And from
such net asset or net remainder off
the couples remaining properties, the
market value at the time of the
marriage will be subtracted and the
resulting totality constitutes the net
profits. Since both H and W have no
separate properties, and nothing
would be returned to each of them,
what will be divided equally between
them are simply the net profits.
However, the trial court forfeited the
half-share of W in favor of her
children. Thus, if Article 102 is used in
the instant case (which should not be
the case), nothing is left to W since
both parties entered into their

marriage without bringing with them


any property.
61.
Given that Article 129 of
the Family Code applies to the
liquidation of the conjugal assets
of H and W, is the latter entitled
to receive any property from the
conjugal partnership?
No. What remains in the
conjugal properties of H and W (after
payment of all debts and obligations)
should be divided equally between
the spouses. However, since W
herself is the guilty party, her share
from the net profits of the conjugal
partnership is forfeited in favor of the
common children pursuant to Article
63(2) of the Family Code. Nothing will
be returned to W because in the
conjugal partnership regime, there is
no separate property which may be
accounted for in the guilty partys
favor. (Quiao v. Quiao G.R. No.
176556, July 4, 2012)
62.
S sells to B a retro a house
and lot for P2.5 million. The
agreement of sale provides that
S cannot repurchase the property
within three years from the date
of the contract. If the sale is
entered into in 2001, when can S
repurchase the property?
S can repurchase the property
within four years from the expiration
of the time within which the right to
redeem cannot be exercised (here
three years from date of the
contract). In other words, S can
repurchase the property until 2008
which is four years from 2004, the
date of the expiration of the time
within which his right to redeem
cannot be exercised. This is pursuant
to paragraph 1 of Article 1606 of the
Civil Code which provides that the

I press toward
BAR OPERATIONS
2013 the mark for the

prize of the high calling of God


in Christ Jesus. - Philippians
3:14

BARRISTERS CLUB

Page 22 of 67

in CIVIL LAW
Prepared by: Atty. Roney Jone P. Gandeza
right of the seller to repurchase the
property sold, in the absence of an
agreement as to the period of
repurchase, shall last four years from
the date of the contract.
63.
Suppose it is stipulated in
the agreement of sale that the
vendor a retro can repurchase
the property whenever he or his
heirs have the means, when can
S repurchase?
S can repurchase the property
within ten years from 2001; i.e., until
2010, if the parties agree on the right
to redeem without specifying the
period of redemption but from the
situation, facts or circumstances, it
can be inferred that the parties
intended a period, the vendor a retro
may deem within ten years from the
date of the property sold within ten
years from the date of the contract.
64.
S offers to B in a letter the
sale of a parcel of land. B sends a
reply. Which of the following
statements in Bs reply will not
result in a contract?
a) I accept your offer to
sell the land. I wish I could
have gotten a better price.
b) I accept your offer to
sell the land, but can you
shave the price?
c) I accept your offer to
sell the land, but only if I
can pay on 90 days credit.
d) I accept your offer to sell
the land, provided that you are
the owner.

shoes assent ot agreement to the


offer. The acceptance must be
unequivocal and communicated to
the offeror. The acceptance in (c) is
not
an
unequivocal
acceptance
because of the condition to pay on
credit; such a condition operates as a
counter-offer.
An
acceptance
may
be
unequivocal even though the offeree
expresses dissatisfaction with the
offer, as in the case of (a) and (b).
The condition in (d) that the
offeror owns the property is implied in
every sale of land, so the condition
does not add any new or different
terms to the offer.
65.
A local civil registrar of a
remote town in Ilocos Sur issued
a marriage license on the same
day that the applicants filed their
application. Will it affect the
validity of the marriage?
No, it will not. The failure of the
local civil registrar to comply with the
publication requirement under Article
17 of the family Code is considered as
a mere irregularity in a formal
requisite of marriage which will not
affect the validity of the marriage, but
would subject the guilty party
administratively, civilly or criminally
liable.
66.
Cindy offers to sell to Daria
a particular car for P300, 000.
Which of the following events will
NOT terminate Cindys offer?
a. A dies prior to Bs
acceptance, and at the time
B accepts, he is unaware of
As death.

(C) is correct. Acceptance is a


voluntary act by the offeree that

I press toward
BAR OPERATIONS
2013 the mark for the

prize of the high calling of God


in Christ Jesus. - Philippians
3:14

BARRISTERS CLUB

Page 23 of 67

in CIVIL LAW
Prepared by: Atty. Roney Jone P. Gandeza
b. The night before B
accepts, fire destroys the
car.
c. B pays 1,000 for a 30-day
option to buy the car.
During this period, A dies,
and later A accepts the
offer, knowing of As death.
d. A dies an hour before
receiving Bs acceptance.
(C) will not terminate Cindys
offer. As a rule, the death of either
the offeror or offeree terminates the
offer, except when the offer is
irrevocable as in the case of an offer
founded upon an option.
The event in (A) will terminate
Cindys offer. The offerees power of
acceptance is terminated when the
offeror of offeree dies or is deprived
of legal capacity to enter into the
proposed contract. An offer is
personal to both parties and cannot
pass to the decedents heirs or
assigns.
The
event
in
(B)
will
automatically terminate Cindys offer
if the specific subject matter of the
offer is destroyed before the offer is
accepted.
(D) will also terminate Cindys
offer for the same reason as in (A).
67.
In which of the following
fact situations would a court
most likely find that an implied
contract existed?
a) X, a noted licensed
physician,
sees
an
unconscious
pedestrian
lying
bleeding
on
the
shoulder of the highway. Z

stops his car, gets out, and


renders emergency medical
care
to
the
injured
pedestrian.
b) X, 82, asks her favorite
niece, Y, a CPA, to fill out
and file his income tax
return. Xs only income is
from this monthly pension
from
SSS
and
interest
income on a bank account.
It takes Y five minutes to
complete the form, takes a
taxi, and goes to personally
file the return at the BIR.
c) X, a contractor, has a
contract to paint Ys house.
X mistakes the house of Ys
neighbor, Z, for Ys house.
As X paints Zs house, Z
stand by watching until the
job is done.
d) A, a homeowner, has
already paid his realty
property tax. A clerk in the
treasurers
office
mistakenly sends A a bill
that should have gone to
As neighbor, B. Being a
good
citizen
and
the
thinking
that
the
treasurers office would not
have sent him the bill if he
did not owe the money, A
pays the bill.
An implied contract existed in
(C). An implied contract is a contract
formed by manifestations of the
parties other than oral or written
language, i.e., by conduct. In this
case, X, by beginning to paint Zs
house, has made an offer by his
conduct because a reasonable person
would conclude that the services
were offered with the expectation of

I press toward
BAR OPERATIONS
2013 the mark for the

prize of the high calling of God


in Christ Jesus. - Philippians
3:14

BARRISTERS CLUB

Page 24 of 67

in CIVIL LAW
Prepared by: Atty. Roney Jone P. Gandeza
compensation
rather
than
gratuitously. Xs offer has been
accepted by Z even though the latter
said nothing.
The situation in (A) does not
give rise to an implied contract
because the pedestrian has not
manifested his consent to the offer
made by X (in contrast to a patient
who goes to Xs office and submits to
treatment by hi,). X is not without a
remedy. He may recover the value of
his services under a quasi-contract. A
quasi-contract is not really a contract
at all; rather, it is a legal fiction
designed to avoid injustice by
preventing unjust enrichment of one
party to the detriment of another.
The situation in (B) does not
create an implied contract. The
conduct of X and Y does not appear
from an objective standard to
manifest contractual intent because
of the close family relationship of the
parties and the minor burden on Y to
render her services. Courts generally
will not presume that a contractual
relationship was intended under
these circumstances.
The situation in (D) likewise
does not create an implied contract.
Without some conduct on the part of
B, such as knowingly accepting the
offered benefits in silence, a court will
not find the manifestation of mutual
consent necessary for an implied
contract.
68.
A
man,
prior
to
his
marriage, made a donation in a
public instrument, in favor of his
future wife, on condition that
should she die before him and
there be no children, one-half of
the properties donated shall be
given to the parents of his wife.

Nine months after the wedding,


the wife died without issue. The
parents now claim the one-half
share given to them in the deed
of donation. Are the parents
entitled to the property?
The parents cannot get the
one-half share. Insofar as said share
is concerned, it cannot be a valid
donation propter nuptias nor a
donation inter vivos nor a donation
mortis causa.
The donation is not a donation
propter nuptias because the share
was not given to one of the spouses.
It is not a donation inter vivos, for
there was no acceptance on the part
of the parents. It is not a donation
mortis causa because the deed of
donation did not have the formalities
of a will, aside from the fact that the
donor is still alive.
69.
Seller is obliged to deliver
to Buyer one of his cars. Is the
object of the sale determinate or
indeterminate?
The object of the sale refers to
a class which in itself is determinate.
Here, the particular thing to be
delivered is DETERMINABLE without
the need of a new contract between
the parties (Art. 1349, CC); it
becomes determinate upon delivery.
70.
What
is
the
mirror
doctrine in the law on sales?
A purchaser of property cannot
close his eyes to facts which should
put a reasonable man on his guard
and claim that he acted in good faith
under the belief that there was no
defect on the vendors title.

I press toward
BAR OPERATIONS
2013 the mark for the

prize of the high calling of God


in Christ Jesus. - Philippians
3:14

BARRISTERS CLUB

Page 25 of 67

in CIVIL LAW
Prepared by: Atty. Roney Jone P. Gandeza
The refusal to believe in the
strong possibility of a defect on the
vendors title will not make the
purchaser an innocent purchase for
value, if circumstances are such that
a reasonably prudent man would
have taken the necessary precaution
if in the same situation. (Embrado v.
CA, 233 SCRA 333)
71.
Apple Computers, Inc. and
a Philippine distributor entered
into an agreement whereby the
distributor agreed to order 1,000
units of Apple Computers every
month and to resell them in the
Philippines at the manufacturers
suggested price plus 10%. All
unsold units at the end of the
year shall be bought back by the
manufacturer at the same price
they
were
ordered.
The
manufacturer
shall
hold
the
distributor free and harmless
from any claim for defects in the
unit.
Is the agreement one of
sale or agency?
The contract is one of agency,
not one of sale. Sale is negated by
the following circumstances:

the price is fixed by the


manufacturer with the 10%
mark-up
constituting
the
commission.

the manufacturer reacquires


the unsold units at exactly the
same price.

warranty for the units was


borne by the manufacturer.

The
foregoing
circumstances
indicate a sale because ownership of

the units was never intended to


transfer to the distributor.
72.
Francisco donated to Cirila
a parcel of land. The donation
was made in a public instrument,
while the acceptance mad by
Cirila was embodied in the same
public instrument. Upon the
death of Francisco, his nephews
and nieces brought an action for
the recovery of the property on
the ground that the done, Cirila,
was the common-law wife of their
uncle at the time of the donation,
and that it is therefore coid
under Article 87 of the Family
Code. Will the action prosper?
Yes. Article 87 of the Family
Code expressly provides that every
donation or grant of gratuitous
advantage,
direct
or
indirect,
between the spouses during the
marriage shall be void, except
moderate gifts which the spouses
may give each other on the occasion
of
any
family
rejoicing.
The
prohibition shall also apply to persons
living together as husband and wife
without a valid marriage. Since
Francisco and Cirilia lived together as
husband and wife without valid
marriage, the inescapable conclusion
is that the donation made by
Francisco in favor of Cirila is void
under Article 87 of the Family Code.
(Arcaba v. Batocael, 370 SCRA 414
[2001])
73.
B buys from S a car,
payable
in
installments.
As
security for the payment of the
balance of the purchase price, a
chattel mortgage is constituted
on the car in favor of S. As
additional
security
for
the
payment of the balance, C, a
friend of B, executes a real

I press toward
BAR OPERATIONS
2013 the mark for the

prize of the high calling of God


in Christ Jesus. - Philippians
3:14

BARRISTERS CLUB

Page 26 of 67

in CIVIL LAW
Prepared by: Atty. Roney Jone P. Gandeza
estate mortgage over his land in
favor of S.

May B enforce against S the


warranty against eviction?

Is the Recto Law applicable


if additional security is given by a
third person who executes a real
estate mortgage to answer for a
deficiency at foreclosure?

Yes. While it appears that B is


evicted by virtue of a right of
subsequent to the sale to him (not
prior to the sale), the warranty may
still be enforced because the cause of
Bs eviction is imputable to the
vendor S.

Yes, because in such a case,


the third person-mortgagor (C), after
paying
the
deficiency
through
foreclosure
of
the
real
estate
mortgage, has the right of a
guarantor who can hold the buyer (B)
liable for the payment made, thus
indirectly violating the Recto Law.
The
Recto
Law
prohibits
foreclosure for such real estate
mortgage because C would be
entitled to proceed against B under
the law on guaranty, thus resulting in
indirect
circumvention
of
the
prohibition.
74.
An agent is allowed to sell
to himself what his principal has
ordered him to buy. Is this
statement accurate?
That statement is inaccurate.
The agent is allowed to sell to himself
what his principal has ordered him to
buy but only with the permission of
his principal. If he does so without
such permission, the sale is void.
75.
S sells a piece of land to B
who does not register the sale.
Subsequently, S sells the same
piece of land to C who registers
the sale. In a suit between the
buyers and B and C over the land
sold, B (the first buyer) is
defeated, and is deprived of the
property.

76.
A offers to sell to B a
particular car for P300, 000.
Which of the following events
creates a contract between A and
B?
a) A dies prior to Bs
acceptance, and at the time
B accepts, he is unaware of
As death.
b) The night before B
accepts, fire destroys the
car.
c) B pays 1,000 for a 30-day
option to buy the car.
During this period, A dies,
and later B accepts the
offer, knowing of As death.
d) A dies an hour before
receiving Bs acceptance.
The event in (C) creates a
contract between A and B because of
the option money which continues to
be effective despite the offerors
death, and despite the offerees
knowledge of such death.
77.
S sold a retro to B a parcel
of
land.
Within
the
period
stipulated for redemption, S
failed to redeem. To register in
the Registry of Property his
consolidation of ownership, B
filed a petition for consolidation,

I press toward
BAR OPERATIONS
2013 the mark for the

prize of the high calling of God


in Christ Jesus. - Philippians
3:14

BARRISTERS CLUB

Page 27 of 67

in CIVIL LAW
Prepared by: Atty. Roney Jone P. Gandeza
but
did
not
name
S
as
respondent. Consequently, S was
not duly summoned and heard.
Has the
jurisdiction?

court

contemplated by law. Such belated


acceptance of Dr. Lim partakes of an
offer by him to Dr. Go which the latter
is free to accept or not.

acquired

No. S should have been named


as a respondent in the case, should
have been summoned, and should
have been heard. The requirement
under Article 1607 of the Civil Code
for a judicial order is not required
for consolidation of ownership of the
property,
but
for
purposes
of
registering the consolidation of title.
78.
Dr. Go offered a partnership
in his private practice to Dr. Lim,
a young surgeon. Dr, Lim, hoping
to secure a more favorable
opportunity, declined Dr. Gos
offer. A few weeks later, Dr. Lim
wrote to Dr. Go saying that he
had reconsidered the matter and
was now accepting the offer
previously made. Did a contract
arise?
a) Yes, because Dr. Gos
offer had no termination
date.
b) Yes, because Dr.
eventually
accepted
offer in writing.

Lim
the

c) No, because Dr. Lims


rejection terminated the
offer.
d) No, because Dr. Lims
refusal
constitutes
a
counter-offer.
(C) is correct because Dr. Lims
rejection terminated Dr. Gos offer. Dr,
Lims belated acceptance of the offer
is not the acceptance of an offer

79. A friend offered to sell Henry


a laptop for P10,000. Henry
replied, saying, Ill pay you
P6,000 now and give you the
balance at the end of the
month. Did a contract arise?
a) Yes, because Henrys
acceptance of the offer was
unconditional.
b)
Yes,
because
the
conditions added by Henry
merely involves the manner
of payment of the purchase
price.
c) No, because the breach
of
the
obligation
is
substantial
enough
that
would
invalidate
the
consent of the offeror.
d)
No
because
the
conditions
of
payment
added by Henry constitutes
a
counter-offer,
thus
invalidating
the
original
offer.
(D) is correct because the
conditions of payment added by
Henry constitute a counter-offer, thus
invalidating the original offer.
80.
What are the steps to be
undertaken by the creditor in
case of non-payment of the debt
on the things pledged?
The steps are:

I press toward
BAR OPERATIONS
2013 the mark for the

prize of the high calling of God


in Christ Jesus. - Philippians
3:14

BARRISTERS CLUB

Page 28 of 67

in CIVIL LAW
Prepared by: Atty. Roney Jone P. Gandeza
(a) The creditor proceeds to a
Notary Public for the sale of the
thing pledged;
(b) The sale shall be made at a
public auction;
(c) The creditor notifies the
debtor and the owner of the
thing pledged of the public
auction stating the amount for
which the public sale is made;
(d) If at the first auction the
thing is not sold, a second one
with the same formalities shall
be held;
(e) If at the second auction
there is still no sale, the
creditor may appropriate the
thing pledged and give an
acquittance of his entire claim.
(Art. 2112, CC)
81.
What
is
the
basis
of
payment of an obligation in case
of extraordinary inflation?
Extraordinary inflation exists
when there is a decrease or increase
in the purchasing power of the
Philippine
currency,
and
such
increase or decrease could not have
been reasonable foreseen or was
manifestly beyond the contemplation
of the parties at the time of the
establishment of the obligation.
82.
What are the rules for the
liquidation
of
the
absolute
community
of
property
or
conjugal partnership of gains in
case of death of a spouse?
The
rules
regarding
the
liquidation of the absolute community
or conjugal partnership are the same.
They are as follows:

1. If a special proceeding for


the settlement of estate of
deceased persons under the
Rules of Court has been
instituted after the death of
one spouse, the absolute
community
or
conjugal
partnership shall be liquidated
in the said proceeding.
2. If no special proceeding for
the settlement of estate of the
deceased spouse is instituted,
the surviving spouse shall
liquidate
the
absolute
community
or
conjugal
partnership either judiciary or
extra-judiciary within one year
from the death of the spouse.
3. If no liquidation is made
within one year from the death
of the deceased spouse, any
disposition or encumbrance
involving any community or
conjugal
property
of
the
terminated marriage shall be
void.
4. Should the surviving spouse
contract
a
subsequent
marriage without liquidating
the community property or
conjugal
partnership,
a
mandatory regime of complete
separation of property shall
govern the property relations
of the subsequent marriage.
This is to protect the heirs of
the deceased spouse. (Art. 103
and 130 FC)
83. When are
extraordinary
applicable?

the

effects of
inflation

The effects of extraordinary


inflation are applicable only where

I press toward
BAR OPERATIONS
2013 the mark for the

prize of the high calling of God


in Christ Jesus. - Philippians
3:14

BARRISTERS CLUB

Page 29 of 67

in CIVIL LAW
Prepared by: Atty. Roney Jone P. Gandeza
there is an official declaration to that
effect by competent authorities.
QUESTIONS
84-85
based on the following
situation:

are
fact

Mayor Alfonso Favis of


Magsingal, Ilocos Sur was on
board a Toyota Fortuner when the
vehicle, cruising at high speed,
accidentally
hit
the
minor,
Marvin,
along
the
national
highway in Vigan City. Despite
medical attention, Marvin died
six days after the accident.
The vehicle was then driven
by Rodel, an employee of the
municipal
government
of
Magsingal, and registered in the
name of Virgilio, a friend of
Mayor Favis.
84.
Is Mayor Favis solidarily
liable for the negligence of the
driver?
Mayor Favis is not liable. The
doctrine of vicarious liability or
imputed liability finds no application
in the present case. Mayor Favis was
neither the employer of Rodel nor the
vehicles registered owner. There
existed
no
causal
relationship
between him and Rodel or the vehicle
used that will make him accountable
for Marvins death. Mayor Favis was a
mere passenger at the time of the
accident.
Parenthetically, it has been
held that the failure of a passenger to
assist the driver by providing him
warnings or by serving as lookout
does not make the passenger liable
for the latters negligent acts. The
drivers duty is not that may be
delegated to others.

85.
Who is liable for Marvins
death?
Liability
attaches
to
the
registered
owner
(Virgilio),
the
negligent driver (Rodel) and the
latters employer (Municipality of
Magsingal). Settled is the rule that
the registered owner of a vehicle is
jointly and severally liable with the
driver for damages incurred by
passengers and third persons as a
consequence of injuries or death
sustained in the operation of said
vehicles. Regardless of whom the
actual owner of the vehicle is, the
operator of record continues to be the
operator of the vehicle as regards the
public and third persons, and as such
is directly and primarily responsible
for the consequences incident to its
operation. (Jayme v. Apostol, G.R. No.
163609, November 27, 2008)
86.
For damage or injuries
arising out of negligence in the
operation of a motor vehicle,
what is the nature of the liability
of the registered owner?
For damage or injuries arising
out of negligence in the operation of
a motor vehicle, the registered owner
may be held civilly liable with the
negligent driver either:
1) subsidiarily, if the aggrieved
party seeks relief based on a
delict or crime under Articles
100 and 103 of the Revised
Penal Code; or
2) solidarily, if the complainant
seeks relief based on a quasidelict under Articles 2176 and
2180 of the Civil Code.

I press toward
BAR OPERATIONS
2013 the mark for the

prize of the high calling of God


in Christ Jesus. - Philippians
3:14

BARRISTERS CLUB

Page 30 of 67

in CIVIL LAW
Prepared by: Atty. Roney Jone P. Gandeza
It is the option of the plaintiff
whether to waive completely the
filing of the civil action, or institute it
with the criminal action, or file it
separately or independently of a
criminal action; his only limitation is
that he cannot recover damages
twice for the same act or omission of
the defendant. (PCI Leasing and
Finance, Inc v. UCPB General
Insurance Co. Inc., G.R. No. 162267,
July 4, 2008)

square meter portion of his lot


sis occupied by Bs house.

87.
D owes C P100, 000. Upon
maturity of the loan, D fails to
pay and so C sues him in a
complaint for sum of money. D
answers
the
complaint
and
before actual hearing, C assigns
the promissory note signed by D
to E for P80, 000. The assignee,
E, now demands payment from D.
For how much is D obliged to pay
E?

No, he is not. Article 448 of the


Civil Code is unequivocal that the
option to sell the land on which
another builds, plants or sows in good
faith, belongs to the landowner. This
advantage in Article 448 of the Civil
Code is accorded the landowner
because his right is older, and by the
principle of accession, he is entitled
to the ownership of the accessory
thing. (Benitez v. C, 226 SCRA 242)

D is liable only for P80, 000,


plus cost and interest. Under Article
1634 of the Civil Code, when a credit
in litigation is sold, the debtor shall
have a right to extinguish it by
reimbursing the assignee for the price
the latter paid therefor, the judicial
costs incurred by him, and the
interest on the price from the day on
which the same was paid. In paying
only such amount, D exercises his
right of legal redemption.

89.
A
owns
a
parcel
of
registered land adjacent to that
of B. A builds a house on his lot.
Unknown to A, however, a portion
of his house has encroached on
Bs
property.
After
the
construction, A sold his lot to C.

(RJPG: The right of redemption


granted in Article 1634 of the Civil
Code is proper only in case of sale [of
the credit in litigation], and not to
cases of barter, donation, or other
modes of acquisition.)
88.
A purchased a property
adjacent to that of B. Shortly
thereafter, A discovered after
relocation survey that a 30-

Despite repeated demands,


B
refused
to
vacate
the
encroached portion. He claims
that under Article 448 of the Civil
Code, he has the pre-emptive
right to purchase the portion
encroached upon.
Is B correct?

Is A a builder in good faith


under Article 448 of the Civil
Code?
Yes, A can be considered a
builder in good faith even if his lot is
covered by a Torrens Title. Good faith
is a state of mind. Unless one is
versed in the science of surveying, no
one can determine the precise extent
or location of his property by merely
examining his paper title.
90.
Is C, As buyer, entitled to
avail of the benefits under Article
448?

I press toward
BAR OPERATIONS
2013 the mark for the

prize of the high calling of God


in Christ Jesus. - Philippians
3:14

BARRISTERS CLUB

Page 31 of 67

in CIVIL LAW
Prepared by: Atty. Roney Jone P. Gandeza
Yes. Upon delivery of the
property
to
him,
C
acquired
ownership of the property and he is
deemed to have stepped into the
shoes of the seller, A, in regard to all
the rights of ownership, including the
right to compel B, the owner of the lot
encroached upon, to exercise either
of the two options granted to him by
law
after
payment
of
proper
indemnity or to sell the portion
encroached upon.
91.
Is B entitled to demand the
removal
of
the
encroaching
structure?
B, the lot owner, cannot
demand
the
removal
of
the
encroaching structure. Such right is
available only if and he chooses to
compel the builder to buy the land at
a reasonable price but the latter fails
to pay it. (Technogas Phil. V. Court of
Appeals, 268 SCRA 5 [1997])
(RJPG:
If
B
decide
to
appropriate the improvements, the
builder has the right to retain the lot
until he is paid of his necessary and
useful expenses. He is not even
required to pay rentals in the
meantime.)
92.
Aragon
is
indebted
to
Benitez and Chua in the amount
of P200, 000. Upon maturity of
the debt, Aragon fails to pay and
so Benitez and Chua sue him in a
complaint for sum of money.
Aragon answers the complaint
and
before
actual
hearing,
Benitez assigns his right to the
credit to Chua (presumably or
P100, 000) for only P75, 000. For
how much is Aragon obliged to
pay Chua?

Aragon is liable to pay P200,


000 to Chua because the assignment
was made to a co-owner. In other
words, Aragon cannot redeem the
credit in litigation sold by Benitez to
Chua. Article 1635 of the Civil Code
enumerates the three instances when
the debtor cannot redeem a credit in
litigation which is sold by his creditor,
one of them being an assignment or
sale to a co-owner.
93.
XYZ Insurance Company
insured Pedros house for Php
500, 000. In the policy, the
insurer undertakes, upon total
loss, to either pay the insure
value of the house, or rebuild it,
upon proof of total loss. If during
the life of the policy the insure
property is completely destroyed,
may the insured insist the
insurance company rebuild his
house than being paid its insured
value?
No, because in alternative
obligations, the right of choice is
given to the debtor, unless it has
been expressly granted to the
creditor (par. 1 Art 1200, Civil Code).
In the absence of an agreement in
the insurance policy giving the right
of choice to the insured, the general
rule applies, and therefore, the
insurer may choose which of the two
prestations
to
perform,
the
performance of one being sufficient.
94.
In 1994, Sonia, a Filipina,
went to work as a nurse in New
York. There she met and fell in
love with Peter, an American,
whom she married in 1996. Sonia
acquired American citizenship in
1998.
Due
to
irreconcilable
differences, however, the couple
parted
ways
in
2000.
The
following year, Peter obtained a

I press toward
BAR OPERATIONS
2013 the mark for the

prize of the high calling of God


in Christ Jesus. - Philippians
3:14

BARRISTERS CLUB

Page 32 of 67

in CIVIL LAW
Prepared by: Atty. Roney Jone P. Gandeza
valid divorce decree in Nevada.
Heartbroken, Sonia repatriated
to the Philippines in 2001 and
reacquired Filipino citizenship
that
same
year.
Sonia
subsequently filed an action
against Peter for declaration of
nullity of marriage under Article
36 of the Family Code with prayer
for support pendent lite. Peter at
that time has settled in the
Philippines with his new wife
Maria, a Filipina.

widow remarried. Is she entitled


to the inheritance?

Will the case prosper?

96.
H instituted his widowed
sister as his heir on condition
that the latter will not marry
again. When H died, his sister
remarried. Is she entitled to the
inheritance?

The case will not prosper. In


Bayot v. Court of Appeals, G.R. No.
155635, November 7, 2008, the
Supreme Court said that the divorce
decree secured by Peter in Nevada
shall be given a res judicata effect in
this jurisdiction. As an obvious result
of the divorce decree obtained, the
marital vinculum between Peter and
Sonia is considered severed; they are
both
free
from
the
bond
of
matrimony. In plain language, Peter
and Sonia are no longer husband and
wife to each other. Consequent to the
dissolution of the marriage, Peter
could no longer be subject to a
husbands obligation under the Family
Code. He cannot, for instance, be
obliged to live with, observe respect
and fidelity, and render support to
Sonia. With the valid foreign divorce
secured by Peter, there is no marital
tie binding him to Sonia. There is on
fine no more marriage to be dissolved
or nullified.
95.
H designated his wife as his
universal
heir
(no
other
compulsory heirs existed) on
condition that when she became
a widow, she must never remarry.
Two years after H died, the

The condition is valid insofar as


the free portion is concerned, since
this absolute prohibition was imposed
by a deceased spouse but is not
valid insofar as her legitimate is
concerned. Therefore, her remarriage
makes her lose the free portion, but
not the legitimate (for ordinarily, no
condition can be imposed on the
legitime. (Art. 874, CC)

Yes, because the condition is


deemed not written. The rule under
Article 874 of the Civil Code is that an
absolute condition not to contract a
first or subsequent marriage shall be
considered as not written unless such
condition has been imposed on the
widow or widower by the deceased
spouse, or by the latters ascendants
or descendants. The condition that
the sister shall not remarry for her to
get the inheritance is void because it
is contrary to good morality and
public policy.
QUESTIONS
97-98
based on the following
situation:

Jessica institutes an action


against Zandro for support in the
allegation that they are husband
and wife and that Zandro has
reneged on his obligation to
financially sipport her as his wife
and Tricia as their child. Zandro
denies that he is married to
Jessica. He avers that their

I press toward
BAR OPERATIONS
2013 the mark for the

prize of the high calling of God


in Christ Jesus. - Philippians
3:14

BARRISTERS CLUB

are
fact

Page 33 of 67

in CIVIL LAW
Prepared by: Atty. Roney Jone P. Gandeza
marriage is void because it was
solemnized without a marriage
license;
that
the
marriage
ceremony was facilitated by an
affidavit wherein they falsely
stated that they had been living
together as husband and wife for
at least five years; and that they
have never cohabited as husband
and wife.
97.
May a party to an action
collaterally attack the validity of
a marriage as what Zandro had
asserted in his answer?
Yes. The court has jurisdiction
to determine the validity of the
marriage of Zandro and Jessica. More
appropriately, the validity of their
void marriage may be collaterally
attacked. Thus, in Nicdao Carino v.
Yee Carino, 403 SCRA Phil. 861
[2001], the Supreme Court declared
that courts are clothed with sufficient
authority to pass upon the validity of
two marriages despite the main case
being a claim for death benefits.
Reiterating Ninal v. Badayog, 384 Phil.
661 [2000], the Supreme Court held
that the courts may pass upon the
validity of marriage even in a suit not
directly instituted to question the
validity of said marriage, so long as it
is essential to the determination of
the case. However, evidence must be
adduced, testimonial or documentary,
to prove the existence of grounds
rendering such a marriage an
absolute nullity.
In the instant case, it is clear
that Zandro and Jessica did not have
a marriage license when they
contracted their marriage. Instead,
they presented a false affidavit
stating that they had been living
together for more than five years. For

this reason, their marriage is deemed


void ab initio.
98.
What is the legal standing
of the false affidavit executed by
Zandro and Jessica?
The falsity of the affidavit
cannot be considered as a mere
irregularity in a formal requisite of
marriage. The law dispenses with the
marriage license requirement for a
man and woman who have lived
together and exclusively with each
other as husband and wife for a
continuous and unbroken period of at
least five years before the marriage.
The false affidavit which Zandro and
Jessica executed so they could push
through with the marriage has no
value whatsoever; it is a mere scrap
of paper. They were not exempt from
the marriage license requirement.
Their failure to obtain a present
marriage
license
renders
their
marriage void ab initio. (De Castro v.
De Castro, GR No. 160172, February
13, 2008)
99.
X obtains a loan from Y.
They agree that upon maturity of
the loan, X will give either the
sum owed or a particular house
and lot. X now assails the
agreement
as
a
pactum
commissorium. Is X correct?
The stipulation is simply an
alternative
obligation,
which
is
expressly allowed by the law. The
agreement to convey the house and
lot in the event of Xs failure to pay
the debt in money at its maturity
does
not
constitute
pactum
commissorium.
It is NOT an attempt to permit
the creditor, Y, to declare forfeiture of
ther security upon the failure of the

I press toward
BAR OPERATIONS
2013 the mark for the

prize of the high calling of God


in Christ Jesus. - Philippians
3:14

BARRISTERS CLUB

Page 34 of 67

in CIVIL LAW
Prepared by: Atty. Roney Jone P. Gandeza
debtor to pay the debt at maturity. It
simply provided that if the debt is not
paid in money, it shall be paid
another way.
100. D borrowed from C P5, 000
payable in one year. When C was
abroad, Cs 16-year old son
borrowed P2, 500 from D for his
school tuition. However, the son
spent the money on a cellular
phone. When the debt to C fell
due, D tendered only P2, 500
claiming compensation on the P2,
500 borrowed by Cs son. Is D
legally entitled to claim partial
legal compensation?
No. This is so because under
Articles 1278 and 1279 of the Civil
Code, in order that there will be a
valid and effective compensation, it is
essential that there must be two
parties who in their own right are
principal creditors and principal
debtors of each other.
In the instant case, C cannot be
considered as a party to the act of his
son in borrowing P2, 500 from D.
Consequently, he did not become a
principal debtor of D; neither did D
become a principal creditor of C.
Therefore, there can be no partial
compensation
of
the
P5,
000
borrowed by D from C.
101. Would the answer be the
same if Cs son actually used the
money for his school tuition?
There would be no difference in
the answer. The fact that Cs son
actually used the P2, 500 for his
school tuition did not make C a party
to the contract between his son and
D. Therefore, C is not the principal
debtor of D and D is not the principal

creditor of C with respect to the


subject amount.
102. A and B are neighbors. On
his buildings wall, A opened a
window beneath the ceiling joists
to admit light in 1995. Even after
10 years, B may still obstruct the
light by constructing on his own
lot a building higher than As
unless
A
makes
a
notarial
prohibition prohibiting B from
making the construction. Of in
23001 A makes the prohibition,
may still B make the obstruction
in 2007?
Yes, because it is only in 2013
(ten
years
after
the
notarial
prohibition) when A may be said to
have acquired the negative easement
of light and view. After 2013, B may
no longer obstruct.
103. About 15 years ago, A
constructed a house on his lot
adjoining the lot owned by B. He
provided
it
with
windows
overlooking Bs lot half a meter
away from the boundary line. A
month ago, B brought an action
against A for the closure of the
windows on the allegation that
they violate the law on distances.
Has
A
acquired
an
easement of light of view by
prescription of ten years?
No. In the first place, there was
no formal prohibition under which A
prohibited B from obstructing his light
and view.
In the second place, A did not
observe the legal requirement that
there should be a distance of at least
two meters between the windows and
Bs lot, since the view is direct.

I press toward
BAR OPERATIONS
2013 the mark for the

prize of the high calling of God


in Christ Jesus. - Philippians
3:14

BARRISTERS CLUB

Page 35 of 67

in CIVIL LAW
Prepared by: Atty. Roney Jone P. Gandeza
According to the Civil Code, nonobservance of this distance does not
give rise to prescription.
104. If the vendee in a contract
of sale expressly renounces the
right to warranty in case of
eviction, and his eviction should
take place, can he still hold the
vendor liable?
It
depends.
In
case
or
renunciation of the warranty without
knowledge of the risks of eviction, the
vendor is only bound to pay the value
of the thing at the time of the
eviction. Although as a consequence
of the waiver, the vendor is not bound
to indemnify the vendee fully in
accordance with Article 1555 of the
Civil Code still the effect of the waiver
cannot be extended as to exempt the
vendor from returning the price.
When eviction occurs, the contract is
left without cause as to the vendee
and inasmuch as his obligation to pay
the price is condition upon the
delivery of the thing by the vendor,
from the moment the vendee is
deprived of the possession of the
thing, the payment of the price really
becomes a payment of what is not
due which, under Article 2154 of the
Civil Code, should be returned.
105. A appoints B to sell his
registered land. B negotiates the
sale with C. Unknown to B, A also
negotiates the sale with D,
thereby
making
contracts
incompatible with each other.
Who is now the owner of the
land, C or D?
Article 1916 of the Civil Code
establishes the rule of preference
when two persons contract with
regard to the same thing, one of them
with the agent and the other with the

principal. Under this provision, that of


prior date shall be preferred, without
prejudice to the provisions of Article
1544 of the Civil Code. Consequently,
whoever between C and D registers
the sale on good faith is the owner of
the land; and in the absence of
registration,
the
first
to
take
possession of the land in good faith.
106. While sitting on his front
porch, Abe watched three men
paint a fence on his property. The
men later learned that they had
made a mistake. The fence they
were to have painted was in the
next block, in the property of
Rey. Is Abe obliged to pay the
painters?
By his toleration of the
trespassers and by his failure to
protest, Abe is obliged to pay the
painters the reasonable value of the
work done and the materials used.
The basis of his liability would be his
implied acceptance of the offer of the
painters to paint his fence. In other
words, the basis of Abes liability is
his implied contract with the painters.
(RJPG: It is error to state that
Abe is liable because of the benefit
bestowed; or that his liability is
grounded on the existence of a quasicontract; or that he is obliged to pay
the painters to prevent unjust
enrichment. This is so because of the
implied contract between him [Abe]
and the painters.)
107. Suppose Abe had not been
at home, and he did not recover
that the fence had been painted
until he returned in the evening,
is Abe liable? If so, what would
be the basis of his liability?

I press toward
BAR OPERATIONS
2013 the mark for the

prize of the high calling of God


in Christ Jesus. - Philippians
3:14

BARRISTERS CLUB

Page 36 of 67

in CIVIL LAW
Prepared by: Atty. Roney Jone P. Gandeza
Abe would not be obliged in
this situation. HE did not learn of the
benefits bestowed until after they had
been completed. There had been no
communication of the offer of the
painters and no express or implied
acceptance by Abe of the work done.
Neither is Abe liable to the painters
under quasi-contract. This is so
because there was neither a case of
colution indebiti or negotiorum gestio
in the instant case.
108. Apolinario
Mabini
died
intestate in 1995, leaving his
wife, Dorothy, four legitimate
children,
and
considerable
properties which they divided
among themselves. Claiming to
be an illegitimate son of the
deceases Apolinario, and having
been left out in the extrajudicial
settlement of Apolinarios estate,
Mario instituted an action for
partition against Dorothy and her
children.
At the trial, Mario admitted
that he had none of the authentic
documents mentioned in Article
172 of the Family Code to show
that he was the illegitimate child
of Apolinario. Is this admission
sufficient basis for Dorothy and
her children to move for the
dismissal of Marios complaint?
Yes. An illegitimate child, like
Mario, is allowed by law to establish
his illegitimate filiation either by an
authentic document or by any other
means allowed by the Rules of Court
and special laws, like his baptismal
certificate, a judicial admission, a
family Bible in which his name has
been entered, common reputation
respecting his pedigree, admission by
silence, testimonies of witnesses, and

other kinds of proof admissible under


Rule 130 of the Rules of Court.
The problem of Mario, however,
is that, since he seeks to prove his
filiation under the second paragraph
of Article 172 of the Family Code, his
action is now barred because of his
alleged fathers death in 1995. This
particular provision of law specifically
provides that when the action to
claim illegitimate filiation is based on
the second paragraph of Article 172,
the action must be brought during the
lifetime of the alleged parent.
It is clear, therefore, that Mario
can no longer be allowed at this time
to introduce evidence of his open and
continuous possession of the status of
an illegitimate child or prove his
alleged filiation through any of the
means allowed by the Rules of Court
or special laws. The simple reason is
that Apolinario is already dead and
can no longer be heard on the claim
of his allged sons illegitimate
filiation. (Uvguanco v. Court of
Appeals. G.R. No. 76873, Ocotber 26,
1989)
109. Rey is a registered guest at
the Pines View Hotel in Baguio
City. In the middle of the night,
Abe, a hotel guest himself, went
up the fire escape, slowly raised
the window in Reys room, went
inside the room, and stole Reys
expensive
laptop.
The
hotel
disclaimed liability on the ground
of force majeure. Is the hotel
liable?
Yes. The problem involves a
case of robbery with force upon
things. Under Article 2001 of the Civil
Code, the act of a thief or robber, who
has entered the hotel is not deemed
force majeure, unless it is done with

I press toward
BAR OPERATIONS
2013 the mark for the

prize of the high calling of God


in Christ Jesus. - Philippians
3:14

BARRISTERS CLUB

Page 37 of 67

in CIVIL LAW
Prepared by: Atty. Roney Jone P. Gandeza
the use of arms
irresistible force.

or

through

an

110. Abe is a bell boy at the


Pines View Hotel. At the point of
a gun, he entered Reys hotel
room and robbed Rey and his
wife of their valuables. The hotel
disclaimed liability on the ground
of force majeure. Is the hotel
liable?
Yes. It is true that here the
robbery was committed with the use
of arms, (which would have been a
case of force majeure) but then Abe
was an employee of the hotel. The
hotel will be liable not because of
force majeure under Article 2001
which evidently refers to a stranger,
but because of Article 2000 of the
Civil Code which provides that the
master is responsible for the acts of
his servants.
111. Arnel pledges his Nokia
cellular phone to Bert to secure a
loan. Arnel and Bert agree that
the latter could purchase the
cellular phone at the current
purchase price if the debt is not
paid on time. Arnel assails the
agreement for being a pactum
commissorium. Is Arnel correct?
No. The stipulation is not a
pactum commissorium. What is
prohibited by Article 2008 of the Civil
Code,
dealing
with
pactum
commissorium, is the automatic
appropriation by the creditor or
pledge of the thing pledged in
payment of the loan at the expiration
of the period agreed upon. Where
there is an express authorization on
the creditor to purchase the thing
pledged at the current market price,
the contract would not come within
the prohibition as there is no

automatic
appropriation
by
the
creditor of pledge of the thing
pledged.
112. Is there an instance under
the law where the creditor is
allowed
to
appropriate
for
himself the thing given by way of
security because of non-payment
of the debt?
Yes, in the case of pledge. If at
the first auction the thing pledged is
not sold, a second auction must be
held. If the thing pledged is not sold
at second auction, the creditor is now
allowed to appropriate for himself the
thing pledged but he must give an
acquittance for his entire claim. (Art.
2112, Civil Code)
113. When the proceeds of the
sale of the mortgaged property in
chattel mortgage does not fully
satisfy the debt, is the mortgage
entitled to recover the deficiency
from the mortgagor?
Yes. It is a settled rule that if
the proceeds of the sale are
insufficient to cover debt either in an
extrajudicial or judicial foreclosure of
mortgage, the mortgagee is entitled
to claim deficiency from the debtor.
While the legislature has denied the
right of a creditor to sue for
deficiency resulting from foreclosure
of security given to guarantee an
obligation as on the case of pledges
(Art. 2115, Civil Code) and in chattel
mortgages of a thing sold on
installment basis (Art. 1484, par. 3,
Civil Code), and the law does not
prohibit
recovery
of
deficiency.
Accordingly, a deficiency claim arising
from the extrajudicial foreclosure of
mortgage is allowed. (PNB v. Court of
Appeals, 308 SCRA 229 [1999])

I press toward
BAR OPERATIONS
2013 the mark for the

prize of the high calling of God


in Christ Jesus. - Philippians
3:14

BARRISTERS CLUB

Page 38 of 67

in CIVIL LAW
Prepared by: Atty. Roney Jone P. Gandeza
114. May
a legitimate
child
impugn his legitimate status?
No, the law itself establishes
the legitimacy of a child conceived or
born during the marriage of his
parents.
The
presumption
of
legitimacy fixes a civil status for the
child born in wedlock, and only the
father (Art. 160, Family Code), or in
exceptional instances the latters
heirs (Art. 171, Family Code), can
contest in an appropriate action the
legitimacy of a child. A child cannot
choose his own filiation.
QUESTIONS 115-116
based on the following
situation:

are
fact

Rey and Mimi were married


in 1985. They were childless. By
stroke of fate, an infant whose
parents
were
unknown
was
entrusted to them by a friend.
Eager to have a child of their
own, Rey and Mimi registered the
child to make it appear that they
were the childs parents. They
named the infant Angela and
followed Reys surname.
The spouses reared and
cared for Angela as if she was
their own. They sent the child to
exclusive schools, and used the
surname of Rey on all her school
records and documents. Rey died
in 1998. In 2000, Mimi married
Abe, an American citizen. Shortly
thereafter, Mimi decided to adopt
Angela
by
availing
of
the
amnesty given under the law to
those individuals who simulated
the birth of a child. Thus, in
2002, Mimi filed a petition for the
adoption of Angela. Abe executed
an affidavit giving his consent to
the adoption of Angela.

115. Can
Angela?

Mimi

alone

Mimi alone cannot adopt


Angela. At the time the petition for
adoption was filed, Mimi had already
remarried. She filed the petition by
herself, without being joined by her
husband, Abe. The law is explicit.
Section 7, Article III of RA No. 8552
specifically provides that the husband
and wife shall jointly adopt, except if
one spouse seeks to adopt the
legitimate child of the other; or if one
spouse seeks to adopt his own
illegitimate child; or if the spouses are
legally separated from each other.
Mimi does not fall under any of
the three exceptions enumerated in
Section 7. First, the child adopted is
not the legitimate child of Mimi or of
her husband Abe. Second, the child is
not the illegitimate child of Mimi. And
third, Mimi and Abe are not legally
separated from each other.
116. Suppose
Abe
filed
for
divorce
during
the
adoption
proceedings, would that make a
difference in you answer?
It will not make any difference
in my answer. The filing of a case for
dissolution of the marriage between
Mimi and Abe is of no moment. It is
not equivalent to a decree of
dissolution of marriage. Until and
unless there is a judicial decree for
the dissolution of the marriage
between Mimi and Abe, the marriage
still subsists. That being the case,
joint adoption by the husband and the
wife is required. To repeat; since at
the time the petition for adoption was
filed, Mimi was married to Abe, joint
adoption is mandatory. (In re: Petition

I press toward
BAR OPERATIONS
2013 the mark for the

prize of the high calling of God


in Christ Jesus. - Philippians
3:14

BARRISTERS CLUB

adopt

Page 39 of 67

in CIVIL LAW
Prepared by: Atty. Roney Jone P. Gandeza
for adoption of Michele P. Lim GR Nos.
168992-93 May 21, 2009)
117. Venus secures a judgment
against Erlinda for the payment
of civil liability arising from the
crime of slander committed by
Erlinda against Venus. To satisfy
the judgment, the sheriff levies
on a real property owned by the
conjugal partnership of Erlinda
and her husband, Romulo. May
the
property
be
sold
on
execution?
The property is exempt from
execution. Article 122 of the Family
Code explicitly provides that payment
of personal debts contracted by the
husband or the wife before or during
the marriage shall be charged to the
conjugal partnership, except insofar
as they redounded to the benefit of
the family. By no stretch of
imagination can it be concluded that
the civil obligation arising from the
crime of slander committed by Erlinda
redounded to the benefit of the
conjugal partnership.
Unlike in the system of
absolute community where liabilities
incurred by either spouse by reason
of a crime or quasi-delict is
chargeable
to
the
absolute
community of the property, in the
absence or insufficiency of the
exclusive property of the debtorspouse, the same advantage is not
accorded in the system of conjugal
partnership of gains. The conjugal
partnership of gains has no duty to
make advance payments for the
liability of the debtor-spouse. (Sps.
Buado v. Court of Appeals, 145222,
April 24, 2009)

118. What is the doctrine of


dependent relative revocation in
the law on succession?
If the testator revokes his will
with the present intention of making
a new one and the new will is not
made, or if made, fails to effect for
any reason whatsoever, it will be
presumed that the testator prefers
the old will to intestacy. The old will
can still be admitted to prbate.
Stated
otherwise,
the
revocation is subject to a SUSPENSIVE
CONDITION: That the testator will
make a new will and that such will
shall take effect. If such condition is
not fulfilled, then there is no
revocation.
119. The Domestic Adoption Act
(RA 8552) requires, inter alia,
that before an alien can adopt in
the Philippines, he must have
resided in the Philippines for at
least three continuous years
prior
to
the
filing
of
the
application for adoption and
maintains such residence until
the adoption decree is entered.
Are there exceptions to this
requirement?
Yes, they are as follows:
(a) The alien is a former Filipino
citizen who seeks to adopt a relative
within the fourth civil degree of
consanguinity;
(b) The alien seeks to adopt
the legitimate or illegitimate child of
his or her Filipino spouse; and
(c) The alien is married to a
Filipino citizen and seeks to adopt
jointly with his or her spouse a
relative within the fourth civil degree

I press toward
BAR OPERATIONS
2013 the mark for the

prize of the high calling of God


in Christ Jesus. - Philippians
3:14

BARRISTERS CLUB

Page 40 of 67

in CIVIL LAW
Prepared by: Atty. Roney Jone P. Gandeza
of consanguinity or affinity of the
Filipino spouse.
120. Abe is the son of Fidel with
his first wife, while Rey is Fidels
son with his second wife. Both
wives predeceased Fidel. Upon
Fidels death in 2008, Abe
immediately instituted and action
for partition of Fidels estate.
After trial in due course, the
court
rendered
judgment
ordering the partition in equal
shares between Abe and Rey the
land exclusively owned by Fidel.
As Abe and Rey failed to
agree on how to partition the
property, the court ordered its
sale at public auction. However,
the public sale did not push
through because Rey refused to
include in the auction sale the
house standing on the land on
the allegation that the house has
been residence nearly 20 years,
and has thus acquired the status
of a family home. Rey also
pointed out that since the house
was not mentioned in Abes
complaint for judicial partition,
such house is not susceptible of
partition. Decide.
The house is deemed included
in the judgment of partition, and this
is true even if its existence was not
mentioned
in
Abes
complaint.
Pursuant to law, since Fidel owned
the land, he also owned the house
which is a mere accessory to the
land. Both properties form part of the
estate of the deceased and are held
in co-ownership by his heirs. Any
decision in the action for partition of
said estate would cover not just the
subject land but also the subject
house.

While it is conceded that the


subject house is covered by the
judgment of partition, the ruling does
not necessarily countenance the
immediate and actual partition of said
property by way of public auction in
the
view
of
the
suspensive
prescription imposed under Article
159 of the Family Code.
Set against the foregoing rule,
the family home consisting of the
subject house and lot on which it
stands- cannot be partitioned at this
time, even if it has passed to the coownership of his heirs. Fidel died in
2008. Thus, for 10 years from said
date or until 2018, or for a longer
period, if there is still a minor
beneficiary residing therein, the
family home he constituted cannot be
partitioned. (Arriola et. al. v Arriola,
GR No. 177703, January 28, 2008)
(RJPG: The Supreme Court
declared in Honrado v. Court of
Appeals, 476 SCRA 280 [2005], that a
claim for exception from execution or
forced sale under Article 153 should
be set up and proved before the sale
of the property at public auction. In
the above-entitled case of Arriola,
Dos timely objected to the inclusion
of the subject house although for a
different reason.
121. Distinguish between right
of redemption and equity of
redemption.
Equity of redemption is the
right of the mortgagor after judgment
in a judicial foreclosure to redeem the
property by paying to the court the
amount of the judgment debt before
the sale or confirmation of the sale.
On the other hand, right of
redemption is the right of the
mortgagor to redeem the property

I press toward
BAR OPERATIONS
2013 the mark for the

prize of the high calling of God


in Christ Jesus. - Philippians
3:14

BARRISTERS CLUB

Page 41 of 67

in CIVIL LAW
Prepared by: Atty. Roney Jone P. Gandeza
sold at an extrajudicial foreclosure
sale by paying to the buyer in the
foreclosure sale the amount paid by
the buyer within one year from such
sale.
122. May the true owner of a
movable
property
recover
possession of his property from
the present possessor? If so, is
there a need to reimburse said
possessor?
The true owner of a movable
property may recover possession of
his property without reimbursement
from a possessor in bad faith or even
from possessor in good faith if said
owner had lost the property or been
unlawfully
deprived
of
it,
the
acquisition being from a private
person.
The owner may also recover
possession of his movable property
but should reimburse the possessor if
such possessor acquired the property
in good faith at a public sale or
auction (Art. 559, CC)
However, the owner can no
longer recover possession of his
movable property, even if he offers to
reimburse, whether or not he had lost
his property or had been unlawfully
deprived of it, if the possessor had
acquired the property in good faith by
purchase from a merchants store, or
in fairs, or markets in accordance with
the Code of Commerce and Special
Laws (Art. 1505, Civil Code and Arts.
85, 86 Code of Commerce); or if the
owner is precluded from denying the
sellers authority, or if the possessor
had obtained the goods because he
was an innocent purchaser for value
and a holder of a negotiable
document title to the goods. (Art.
1518, CC)

123. Why is a negative easement


acquirable
by
prescription
despite its non-apparent?
Generally, negative easements
cannot be acquired by prescription
because they are non-apparent. Still,
the very existence of the Civil Code
(insofar as it relates to negative
easements), prove that in certain
cases,
and
for
purposes
of
prescription, there are negative
easements that may be indeed be
considered apparent, not because
there are visible signs of existence
but because of the making a notarial
prohibition. The notarial prohibition
makes apparent what really is nonapparent.
124. While
walking
along
Session Road in Baguio City, A
found a purse containing P50,
000. Can A become the owner of
the P50, 000?
If A knows the owner of the
purse, there is no way by which he
can become the owner of the P50,
000 because under the law, he must
return the purse, including its
contents, to the owner.
If the owner is unknown, A
shall
immediately
deposit
the
movable with the mayor of the place
where the purse was found. There
shall then be a public announcement
of the finding for two consecutive
weeks.
Six
months
from
the
publication having elapsed without
the owner having appeared, the thing
found shall be awarded to A, after
reimbursement of the expenses. (Art.
719, CC) Should the owner appear in
time, he shall be obliged to pay A, as
a reward, 1/10 of the amount found.
(Art. 720, CC)

I press toward
BAR OPERATIONS
2013 the mark for the

prize of the high calling of God


in Christ Jesus. - Philippians
3:14

BARRISTERS CLUB

Page 42 of 67

in CIVIL LAW
Prepared by: Atty. Roney Jone P. Gandeza
125. When can the owner of an
estate claim a compulsory right
of way?
Under Articles 649 and 650 of
the Civil Code, the owner of an estate
may claim a compulsory easement of
right of way after he has established
the existence of the following
requisites:
(a)
the
estate
is
surrounded by other immovables and
is without an adequate outlet to a
public highway; (b) proper indemnity
is paid; (c) the isolation is not due to
the proprietors own acts; and (d) the
right of way claimed is at a point least
prejudicial to the servient estate and
insofar as consistent with the law,
where
the
distance
from
the
dominant estate to a public highway
may be the shortest.
126. May an existing easement
of right of way be extinguished
by the opening of an adequate
outlet to a public highway?
An easement of right of way
provided for in a contract of sale is a
voluntary easement. As such, it
cannot be extinguished by the
opening of an adequate outlet to a
public highway. The opening of an
adequate outlet can extinguish a
legal or compulsory easement but not
a voluntary easement. (La Vista
Association v. Court of Appeals, 278
SCRA 498 [2000])
127. B donated to C a parcel of
land subject to a condition. When
C failed to comply with the
condition, B sold the land to D. Is
the sale an act of revocation of
the donation?
No. The act of selling the
property
donated
cannot
be
considered as a valid act of

revocation of the donation because a


formal action in court to revoke the
donation must be filed by the donor
pursuant to Article 764 of the Civil
Code which speaks of an action that
has a prescriptive period of four (4)
years from non-compliance with the
condition stated in the donation. The
rule that there can be automatic
revocation without the benefit of a
court action does not apply in this
case because the subject donation is
devoid of any provision providing for
automatic revocation in the event of
con-compliance with the condition set
forth therein. Thus, a court action is
necessary to be filed within four (4)
years from the non-compliance with
the condition violated.
128. May the heirs of the donor
sue the heirs of the done for
revocation of the donation if
there is a violation of any
restriction
in
the
deed
of
donation?
Yes. Under Article 764 of the
Civil Code, the donor or his heirs have
the personality to question the
violation of any restriction in the deed
of donation. Consequently, the right
to revoke be transmitted to the heirs
of the donor and may be exercised
against the heirs if the done, and the
action prescribes after four years
from the violation of the condition.
129. A lease agreement contains
the following stipulation: That
the term of the lease shall be 14
years beginning April 1, 1998 and
may be renewed for a like term at
the option of the lessee. The
lessor
contends
that
this
stipulation is void because it is
violative if the principle of
mutuality of contracts under

I press toward
BAR OPERATIONS
2013 the mark for the

prize of the high calling of God


in Christ Jesus. - Philippians
3:14

BARRISTERS CLUB

Page 43 of 67

in CIVIL LAW
Prepared by: Atty. Roney Jone P. Gandeza
Article 1308 of the Civil Code. Is
the lessor correct?
No. The fact that the lessees
option to renew the lease is binding
only on the lessor and can be
exercised only by the lessee does not
render such option void for lack of
mutuality. After all, the lessor is free
to give or not to give the option to the
lessee. And while the lessee has a
right to elect whether to continue
with the lease or not, once he
exercises his option to continue and
the lessor accepts, both parties are
thereafter bound by new lease
agreement.
Their
rights
and
obligations become mutually fixed,
and the lessee is entitled to retain the
possession of the property for the
duration of the new lease, and the
lessor may hold him liable for the rent
thereof. Mutuality obtains in such a
contract and equality between the
lessor and the lessee since they
remain with the same faculties in
respect to fulfillment.
130. How shall the clause may
be renewed for a like term at the
option
of
the
lessee
be
interpreted or applied?
The clause means that the
exercise by the lessee of his option
resulted in the automatic extension of
the contract of lease under the same
terms and conditions prevailing in the
original contract of lease, i.e., for 14
years, the phrase for a like term
referring to the term of the lease. If
the renewed contract were still
subject to mutual agreement by the
lessor and the lessee, then the option
which is an integral part of the
consideration for the contract would
be rendered worthless. For then, the
lessor could easily defeat the lessees
right
by
simply
imposing

unreasonable and onerous conditions


to prevent the parties from reaching
an agreement. (Allied Banking Corp.
v. Court of Appeals, 284 SCRA 357
[1998])
131. May a lessee invoke the
application of Article 16887 of
the Civil Code for the extension
of
his
lease
in
the
same
ejectment case filed by his
lessor?
Yes. The exercise of the power
given to the courts in Article 1687 of
the Civil Code to extend the period of
the lease when the lessee has been in
occupancy of the premises for more
than a year does not contemplate a
separate action for that purpose. That
power may be exercised as an
incident in the action for ejectment
itself and by the court having
jurisdiction over it.
132. May a common carrier be
held liable for the death or injury
of a passenger caused by the
willful acts or negligence of
another passenger or stranger?
Yes, a common carrier is
responsible for the death or injury of
a passenger caused by the willful act
or negligence of another passenger or
stranger, but only if the employees of
the common carrier through the
exercise of diligence of a good father
of family could have prevented the
act or omission.
133. When does an action for
reconveyance based on implied
or constructive trust prescribe?
An action for reconveyance of a
parcel of land based on an implied or
constructive trust prescribes in ten
years, the point of reference of being

I press toward
BAR OPERATIONS
2013 the mark for the

prize of the high calling of God


in Christ Jesus. - Philippians
3:14

BARRISTERS CLUB

Page 44 of 67

in CIVIL LAW
Prepared by: Atty. Roney Jone P. Gandeza
the date of registration of the deed or
the date of issuance of the certificate
of title over the property. However,
this rule applies only when the
plaintiff or the person enforcing the
trust is not in possession of the
property, since if a person claiming to
be the owner thereof is in actual
possession of the property, the right
to seek reconvenyance which in
effect seeks to quiet title to property,
does not prescribe.
134. A contract of lease contains
a stipulation authorizing the
lessor to take over possession of
the leased premises without
judicial intervention upon failure
of the lessee to pay rent. Is the
stipulation valid?
The validity of a contractuallystipulated termination clause has
been upheld by the Supreme Court.
The stipulation is in the nature of
resolutory condition, for upon the
exercise by the lessor of his right to
take possession of the leased
property, the contract is deemed
terminated. This kind of contractual
stipulation is not illegal, there being
nothing in the law proscribing such
kind of agreement.
Moreover, judicial permission
to cancel the lease agreement is not,
therefore, necessary because of the
express stipulation in the contract of
lease that the lessor, in case of failure
of the lessee to comply with the
terms and conditions thereof, can
take over the possession of the
leased premises, thereby cancelling
the contract of lease. Resort to
judicial action is necessary only in the
absence of a special provision
granting the power of cancellation.

135. D borrowed from C P350,


000.00 payable in three months.
To secure payment of the loan, D
executed in Cs favor a Real
Estate Mortgage. From the loan
amount that D was to receive,
P17, 500.00 was pre-deducted as
interest for the first month which
was equivalent to 5% of the
principal debt, and P7, 500.00
equivalent to 1.79% of the debt,
was further deducted as service
fee. Thus, D actually received
only P325, 000.
When D failed to pay the
loan, the loan was extended
through
a
restructuring
agreement in such a way that the
unpaid interest became part of
the principal. The restructuring
agreement adopted all other
terms and conditions contained
in the first loan agreement.
Due
to
the
continued
inability of D to pay, the loan
agreement was renewed to five
more
times.
In
all
these
subsequent renewals, the same
terms and conditions found in the
first agreement were retained.
Consequently, the principal was
finally increased to P880, 000.
This increase in the amount of
principal had been due to unpaid
interest and other charges.
When the debt remained
unpaid, C formally demanded
from D the payment of P880, 000.
When the demand was ignored, C
foreclosed the mortgage. At the
auction sale, the mortgaged
property was sold to C, the lone
bidder, for the amount of P880,
000.
Is the foreclosure sale
valid? What is Ds remedy in law?

I press toward
BAR OPERATIONS
2013 the mark for the

prize of the high calling of God


in Christ Jesus. - Philippians
3:14

BARRISTERS CLUB

Page 45 of 67

in CIVIL LAW
Prepared by: Atty. Roney Jone P. Gandeza
The foreclosure is void. Since D
was not given an opportunity to settle
his debt, at the correct amount and
without
the
iniquitous
interest
imposed, no foreclosure proceedings
may be instituted. A judgment
ordering
a foreclosure
sale
is
conditioned upon a finding on the
correct amount of the unpaid
obligation and the failure of the
debtor to pay the said amount. (Secs.
2 and 3, Rule 68, Rules of Court)

for reconveyance. Since the property


has not yet been transferred by C to
an innocent purchase for value, D
may still avail of the remedy of
reconveyance. (Espiritu v. landrito,
GR Nos. 169617, April 4, 2007)
136. Donor donates on January
1. Donee accepts on January 5.
Donor
dies
on
January
8.
Acceptance of donation reached
donors house on January 10. Is
there a perfected donation?

In this case, it has not yet been


shown that D had already failed to
pay the correct amount for the debt,
and therefore, a foreclosure sale
cannot be conducted in order to
answer for the unpaid debt. The
foreclosure sale is conducted upon
Ds failure to pay P880, 000 is void
since the amount demanded as the
outstanding loan was overstated,
consequently, it has not been shown
that D has failed to pay his
outstanding obligation.

There is no perfect donation because


done died at the time of the making
of the donation.

As a result, the subsequent


registration of the foreclosure sale
cannot transfer any rights over the
mortgaged property to C. The
registration of the foreclosure sale,
which is void, cannot vest title over
the mortgaged property. The torrens
system does not create or vest title
where one does not have a rightful
claim over a real property. It only
confirms and records title already
existing and vested. It does not
permit one to enrich himself at the
expense of the another. Thus,
registration or real property by one
person in his or her name, whether by
mistake or fraud, the real owner
being another person, impresses
upon the title acquired the character
of a constructive trust for the real
owner, which would justify an action

(RJPG: The term making


means perfection of the donation;
otherwise,
if
making
means
giving, Article 737 of the Civil Code
would be inconsistent with Article 734
which states that the donation is
perfected from the moment the donor
knows of the acceptance by the
done. Thus, at the time the donation
is perfected, both the donor and the
done must be capacitated.)

137. Insane donor donates on


January 1. Donee accepts on
January 5. Donor becomes san on
January
7.
Donor
receives
acceptance on January 8. Is there
a perfect donation?
Yes, because at the time of the
making, both parties had capacity.

138.
A Real Estate Mortgage
contains a stipulation that the
mortgagee shall send notice of
foreclosure proceedings to the
mortgagor at the latters given
address.
If
the
mortgagee
decides
to
extrajudicially
foreclose
the
mortgage,
is

I press toward
BAR OPERATIONS
2013 the mark for the

prize of the high calling of God


in Christ Jesus. - Philippians
3:14

BARRISTERS CLUB

Page 46 of 67

in CIVIL LAW
Prepared by: Atty. Roney Jone P. Gandeza
personal notice to the mortgagor
still necessary?

repurchase
prescribed.

Yes. In extrajudicial foreclosure


proceedings, personal notice to the
mortgagor is actually unnecessary
unless stipulated. In this case, the
mortgagor
and
the
mortgagee
voluntarily agreed on an additional
stipulation
embodied
in
their
mortgage agreement. Not being
contrary to law, morals, good
customs, public order or public policy,
the mortgagee should send notice of
the extrajudicial proceedings to the
mortgagor in compliance with the
stipulation in the mortgage deed.
(Tamayo, Jr. v. Heirs of Gavino
Dominguez, 498 SCRA 342 [2006])

139. Is B entitled to repurchase


the subject property?

QUESTIONS 139-141
are
based on the following situation:
In 1976, A obtained an
original
certificate
of
title
covering a parcel of land secured
through a homestead patent.
Upon As death in 1978, the land
was transferred by succession to
his son B who obtained a transfer
certificate of title in his name.
In 1989, B mortgaged the
land to DBP as security for a
loan. When B failed to pay, the
bank extrajudicially foreclosed
the mortgage, purchased the
property at the public auction,
and secured a title in its name
after consolidation.
Invoking Section 119 of the
Public Land Act, B tried to
repurchase the property in 2002,
but the bank refused. The bank
defends that there can no longer
be any right of repurchase but by
a transfer certificate of title, and
that in any event, the right to

had

Yes, B is entitled to repurchase.


The plain intent of Section 110 of the
Public Land Act is to give a
homesteader
every
chance
to
preserve and keep in the family the
land that the State has gratuitously
given him as a reward for his labor in
cleaning, developing and cultivating
it. Hence, the fact that the land had
been inherited by the patentees son
(and a new titled in his name is
issued) does not bring it outside the
purview of Section 119. In fact, the
policy behind the law is fulfilled
because the land remains in the
family of the patentee.
140. Is Bs right to repurchase
already time-barred?
The rules on redemption in
case of an extrajudicial foreclosure of
land acquired under free patent or
homestead
statutes
may
be
summarized as follows: If the land is
mortgaged to parties other than rural
banks, the mortgagor may redeem
the property within one (1) year from
the registration of the certificate of
sale pursuant to Act No. 3135. If he
fails to do so, he or his heirs may
repurchase the property within five
(5) years from the expiration of the
redemption period also pursuant to
Section 119 of the Public Land Act.
Consequently, the complaint filed in
2002 was on time.
141. If B is already deceased,
may his widow exercise the right
of redemption?

I press toward
BAR OPERATIONS
2013 the mark for the

prize of the high calling of God


in Christ Jesus. - Philippians
3:14

BARRISTERS CLUB

already

Page 47 of 67

in CIVIL LAW
Prepared by: Atty. Roney Jone P. Gandeza
The term legal heirs as used
in Section 199 is used in generic
sense. It is broad enough to cover any
person who is called to the
succession either by provision of a
will or by operation of law. Thus, legal
heirs include both testate and
intestate
heirs depending upon
whether succession is by will of the
testator or by law. Legal heirs are not
necessarily compulsory heirs but they
may be so if the law reserves a
legitime for them. The interpretation
of legal heirs is more in keeping
with the salutary purpose behind the
enactment of Section 119 and the
jurisprudence laid down on the
matter. Indeed, it is not far-fetched to
arrive at a more liberal conclusion if
the section is analyzed in accordance
with its purpose.

both the donor and the donee must


be capacitated.)

The
widow
inherited
the
property from B, her husband, who in
turn inherited it from his father A. The
widow, as daughter-in-law of the
patentee, can be considered as
among the legal heirs who can
repurchase the land. The Supreme
Court has time and again said that
between
two
statutory
interpretations, that which better
serves the purpose of the law should
prevail. Furthermore, the law must be
liberally construed in order to carry
out its purpose. (DBP v. Gagarani, GR
No. 172248, September 17, 2008)

143. Article 75 of the Family


Code specifically provides that in
the absence of a marriage
settlement, or when the regime
agreed upon is void, the system
of
absolute
community
of
property
shall
govern
the
property relations of the spouses
during their marriage. Is this rule
absolute?

(RJPG: The term making must


be interpreted to mean perfection
of
the
donation;
otherwise,
if
making means giving, Article 737
of
the
Civil
Code
would
be
inconsistent with Article 734 which
states that the donation is perfected
from the moment the donor knows of
the acceptance by the donee. Thus,
at the time the donation is perfected,

142. Y, the prospective husband,


gives a donation propter nuptias
of a parcel of land to X, his
prospective wife. Due to quarrel,
the marriage is called off. Is the
donation automatically revoked?
I distinguish. If the donation
propter nuptias is incorporated in X
and Ys settlement, the donation is
rendered void by the non-celebration
of the marriage (Art. 81, Family
Code). However, if the donation is
propter
nuptias
is
made
independently
of
the
marriage
settlement, the donation is revocable
only at the instance of the donor.
(Art. 86 (1), Family Code)

The rule is not absolute.


Pursuant to Article 103 of the family
Code, should the surviving spouse
contract a subsequent marriage
without liquidating the absolute
community of property regime or
conjugal partnership, a mandatory
regime of complete separation of
property shall govern the property
relations of the subsequent marriage.
This is to protect the heirs of the
deceased spouse.
144. What unions are covered by
Articles 147 and 148 of the
Family Code?

I press toward
BAR OPERATIONS
2013 the mark for the

prize of the high calling of God


in Christ Jesus. - Philippians
3:14

BARRISTERS CLUB

Page 48 of 67

in CIVIL LAW
Prepared by: Atty. Roney Jone P. Gandeza
Article 147 applies to two
relationships. The first is when a man
and a woman who are capacitated to
marry each other live exclusively as
husband and wife without the benefit
of marriage. The second is when a
man and a woman live together
under a void marriage where the
parties do not have an existing
marriage with another.
Article 148, on the other hand,
applies to five kinds of relationships;
namely (1) bigamous marriages; (2)
adulterous
relationships;
(3)
relationships
in
a
state
of
concubinage;
and
(5)
multiple
alliances of the same man.
145. X has no child. At the time
he gave a donation of P100, 000,
he had P1 million. Therefore,
after the donation, he had P900,
000 left. Later he adopted a
minor child. At the time he made
the adoption, he had only P50,
000 left. Should the donation be
reduced? How much and within
what period?
The
donation
should
be
reduced by P25, 000 because the
legitime is impaired to that extent.
The action to revoke or reduce the
inofficious donation must be brought
within four years from adoption. Thus:
50, 00
(value at the time of
adoption)
100,000 (value of donation)
150,000
The adopted child has the
same rights as a legitimate child. As
such, he is entitled to a legitimate of
P75, 000. But because the residue of
Xs estate is only P50, 000, the
donation is reducible by P25, 000.

146. Explain the doctrine of selfhelp in civil law.


The doctrine of self-help in civil
law is the right to counter in certain
cases, forces with force. It is
enunciated under Article 429 of the
Civil Code, and it can be exercised by
the owner at the time of actual or
threatened dispossession. When the
possession has already been lost, the
owner must resort to judicial process
for the recovery of property.
147. Is the rule on accession
discrete absolute?
The rule is not absolute. The
following are the instances when the
owner has no right to the fruits of his
property:
(a)
If the thing is in the possession
of a possessor in good faith, in which
case possessor is entitled to the
fruits. (Art. 554, CC)
(b)
If the thins is subject to a
usufruct,
in
which
case
the
usufructuary is entitled to the fruits.
(Art. 556, CC)
(c)
If the thing is leased, in which
case the lessee is entitled to the fruits
leased, although such lessee must
pay to the owner rentals which are in
the nature of civil fruits. (Art. 1654,
CC)
(d)
If the thing is in the possession
of an antichretic creditor, in which
case such creditor is entitled to the
fruits with the obligation of applying
them to the interest and principal.
(Art. 2132, CC)
148. What are the rules in
adjunction?
Adjunction is governed by the
following rules:

I press toward
BAR OPERATIONS
2013 the mark for the

prize of the high calling of God


in Christ Jesus. - Philippians
3:14

BARRISTERS CLUB

Page 49 of 67

in CIVIL LAW
Prepared by: Atty. Roney Jone P. Gandeza
(a)
If the two things can be
separated
without
injury,
their
respective owners may demand their
separation. (Art. 469, par. 1 Civil
Code)
(b)
If the two things cannot be
separated without injury, and both
the owners had acted in good faith,
the owner of the principal thing
acquires the accessory indemnifying
the owner of such accessory for its
value
(Art.
456,
Civil
Code).
Nevertheless, if the accessory thing is
much more precious than the
principal thing, the owner may
demand its separation, even though
the principal thing may suffer some
injury.
(c)
If the owner of the accessory
thing has made the incorporation in
bad faith, he loses the thing
incorporated and shall have the
obligation to indemnify the owner of
the principal thing for the damages
which the latter may have suffered.
(d)
If the one who has acted in bad
faith is the owner of the principal
thing, the owner of the accessory
thing may choose between the
former paying him its value or that
his accessory thing be separated,
even though it will cause damage or
injury to the principal. Moreover, the
owner of the principal thing shall be
liable for damages.
(e)
If both owners had acted in bad
faith, their respective rights shall be
determined as though both had acted
in good faith.
149. What is meant by rebus sic
stantibus?

Rebus sic stantibus is a rule


which provides that an agreement is
valid only if the same conditions
prevailing at the time of contracting
continue to exist at the time of
performance.
150. What is the nature of
reformation of instruments as a
remedy in law?
Reformation of instruments
(not reformation of contracts) is a
remedy to conform to the real
intention of the parties due to
mistake, fraud, inequitable conduct,
or accident. The action prescribed in
10 years from date of execution of
instrument.
151. What
nuisance?

is

an

An attractive nuisance is a
dangerous
instrumentality
or
appliance which is likely to attract
children at play.
152. What is the doctrine
attractive nuisance?

of

One who maintains on his


estate or premises an attractive
nuisance without exercising due care
to prevent children from playing
therewith or resorting thereto, is
liable to a child of tender years who is
injured thereby, even if the child is
technically a trespasser in the
premises. The reason for the doctrine
is that the attractiveness is an
invitation to children. Safeguards to
prevent danger must be set up.
153. X
was
coerced
into
marrying
Y.
X
sued
for
annulment. During the pendency
of the case, X married Z. When Z
learned of the first marriage, Z

I press toward
BAR OPERATIONS
2013 the mark for the

prize of the high calling of God


in Christ Jesus. - Philippians
3:14

BARRISTERS CLUB

attractive

Page 50 of 67

in CIVIL LAW
Prepared by: Atty. Roney Jone P. Gandeza
sued X for bigamy. X now alleges
that
the
pendency
of
the
annulment case is a prejudicial
question. Is X correct?
X is wrong because the
decision in the annulment case is not
important. The first marriage will
either be annulled or not. If not
annulled, bigamy can prosper. If
annulled, still bigamy can prosper, for
when he married the second time, he
was still married to his wife, a
voidable marriage being considered
valid until annulled.
154. X, a married man, was
forced by Y to contract marriage
with her. X then sued for
annulment
of
the
second
marriage; Y retaliated with a
charge of bigamy. In the bigamy
case, X moved to suspend the
criminal proceedings until after
the termination of the annulment
case on the ground that the
annulment case is a prejudicial
question. Should the motion be
granted?
Yes, because the annulment
case poses a prejudicial question.
If X was really forced into
marrying Y, then his consent was
defective;
hence,
the
second
marriage is to be annulled on that
ground. He cannot therefore be guilty
of bigamy.
155. In his will, testator T a)
disinherits
his
daughter,
A,
because she married a food for
nothing
gigolo
despite
my
repeated
warnings
that
she
shouldnt marry him; b) omits
his wife, W; c) leaves a legacy of
P10, 000 to his mistress, M, and
P5, 000 to his driver, E; and d)

institutes his son, B, as his sole


heir. Distribute T;s estate of
P100, 000.
The disinheritance of A was
ineffective because the ground relied
upon by T does not constitute a valid
ground for disinheritance under
Article 919 of the Civil Code. Hence,
the testamentary provisions in the
will shall be annulled but only to the
extent that As legitime was impaired.
The total omission of W does
not constitute preterition because she
is not a compulsory heir in the direct
line. Only compulsory heirs in the
direct line may be the subject of
preterition.
Not
having
been
preterited, she is entitled to her
legitime.
The legacy in favor of M is void
under Article 1028 of the Civil Code
for being in consideration of her
adulterous relations with T. She is,
therefore, disqualified to receive the
legacy of P10, 000.
The legacy of P5, 000 in favor
of E is not inofficious because it does
not exceed the free portion. Hence, E
shall be entitled to receive it.
The institution of B, which
applies only to the free portion, shall
be respected.
In sum, the estate of T will be
distributed as follows:
A
B
W
E
M

25, 000
45, 000
25, 000
5, 000
0

I press toward
BAR OPERATIONS
2013 the mark for the

prize of the high calling of God


in Christ Jesus. - Philippians
3:14

BARRISTERS CLUB

Page 51 of 67

in CIVIL LAW
Prepared by: Atty. Roney Jone P. Gandeza
156. Within what period may an
action to revoke a donation inter
vivos be filed by the donor?
An action to revoke a donation
inter vivos must be filed by the dono
within four years if the ground is the
subsequent
birth,
adoption
or
reappearance of a child of the donor
or non-fulfillment of a condition; and
it must be filed within one year id the
ground is act of ingratitude in the part
of the donee. As to the first two
grounds, the right to file the action is
transmitted to the heirs; as to the last
ground only the donor can file the
action and the right is not transmitted
to the heirs.
157. What
title?

is

muniment

of

A muniment of title is an
instrument or written evidence which
an application for land registration
holds or possesses to enable him to
substantiate and prove title to his
estate.
158. When
is
pendens
not
applicable?

a
notice
registrable

lis
or

A notice lis pendens is not


registrable or applicable to the
following: (a) preliminary attachment;
(b) proceedings for the probate of
wills; (c) levies on execution; (d)
proceedings for administration of
estate of deceased persons; and (e)
proceedings in which the only object
is the recovery of sum of money.
159. Does
a
previous
final
judgment denying a petition for
declaration of nullity of marriage
on the ground of psychological
incapacity bar a subsequent
petition for annulment on the

ground of
license?

lack

of

Yes. In both petitions, the


cause of action is the same the
declaration of nullity of marriage.
What differs is only the ground upon
which
the
case
of
action
is
predicated. A party cannot avoid the
application of res judicata by simply
varying the form of his action or
adopting a different method of
presenting his case. (Mallion v.
Alcantara, GR No. 141528, October
31, 2006)
160. Articles 238 to 253 of the
Family Code govern Summary
Judicial Proceedings in the Family
Law. Are these proceedings
governed by the Revised Rule on
Summary Procedure?
No. These summary judicial
proceedings fall within the jurisdiction
of the RTC, while the cases covered
by the Revised Ruled on Summary
Procedure are cases falling within the
jurisdiction of the Municipal Trial
Court.
These
summary
judicial
proceedings are in fact governed by
the special rules in Articles 239 to
253 of the Family Code. Thus, in the
proceedings to secure the consent of
the estrange spouse to a transaction,
a preliminary conference is to be
conducted by the judge without the
parties being assisted by counsel;
while under the Revised Rules on
Summary Procedure, the parties who
are required to be present in the
preliminary conference, are assisted
by counsel.
161. A
person
who
is
a
beneficiary in a will is competent
to act as an instrumental witness
in the execution of a will. Is this
statement accurate?

I press toward
BAR OPERATIONS
2013 the mark for the

prize of the high calling of God


in Christ Jesus. - Philippians
3:14

BARRISTERS CLUB

marriage

Page 52 of 67

in CIVIL LAW
Prepared by: Atty. Roney Jone P. Gandeza
Yes, but his institution as an
heir, or the legacy or devise given to
him, shall be rendered void, unless
there are three other competent
witnesses (Art. 823, CC). In other
words, he is disqualified from
inheriting from the testator (Art.
1027, CC).
162. If a marriage is dissolved
because of the death of the
husband, what surname may the
widow use?
Although the death of the
husband dissolves the marriage ties,
still the window may desire to cherish
her deceased husbands memory by
the continued use of his surname.
However, if she does not want to, she
is allowed to use her maiden
surname. Notice the use of the word
may in Article 373 of the Civil Code.
163. If a marriage is annulled, is
the wife required to resume her
maiden name and surname?
Wife is the guilty spouse: She
SHALL resume her maiden name and
surname.
Wife is the innocent spouse:
She MAY resume her maiden name
and surname, but she may choose to
continue
employing
her
former
husbands surname, unless (1) the
court decrees otherwise; or (2) she or
the former husband is married again
to another person. (Art. 371, CC)
164. What if legal separation
occurs, is the wife entitled to
continue using the husbands
surname even of she is the guilty
spouse?

Article 372 of the Civil Code


does not distinguish whether the wife
is the guilty spouse or not, unlike in
the case of annulment of marriage
under Article 371, because in legal
separation the marriage ties still
subsist.
165. D owes C P30, 000. D goes
to C and tells the latter that he is
proposing X as the new debtor
who will pay the debt. C accepts
the substitution. If X later
becomes
insolvent,
may
C
demand
payment
of
the
obligation from D?
What obtains in this problem is
delegacion. In delegacion, the debtor
offers, and the creditor accepts, a
third person who consents to the
substitution. Pursuant to Article 1295
of the Civil Code, the insolvency of X
who has been proposed by D and
accepted by C, shall not revive Cs
action against D, the original obligor,
except when said insolvency already
existed and of public knowledge, or
known to the debtor when he
delegated his debt.
166. Suppose the insolvency of
X is of public knowledge but not
known to D and the creditor who
also does not know, gives his
consent, may C hold D liable?
Yes, because the initiative
came from the debtor who should
know the economic or financial status
of his substitute or delegado.
167. If an agreement falling
within the Statute of Frauds is
made
in
writing
and
duly
subscribed but the same is
subsequently lost or destroyed,
is
the
agreement
judicially
enforceable?

I press toward
BAR OPERATIONS
2013 the mark for the

prize of the high calling of God


in Christ Jesus. - Philippians
3:14

BARRISTERS CLUB

Page 53 of 67

in CIVIL LAW
Prepared by: Atty. Roney Jone P. Gandeza
Yes,
because
the
first
(introductory) part of the Statute of
Frauds (Art. 1403, par. 2 Civil Code)
states that if the agreement is not in
writing and duly subscribed, evidence
of such agreement cannot be
received without the writing, or a
secondary evidence of its contents.
Thus, the loss or destruction of a
written agreement (complying with
the Statute of Frauds) will not militate
against the validity or enforceability
of the agreement. The agreement us
valid and enforceable and its
existence and the contents thereof
can be proved by secondary evidence
like the testimony of the interested
part and his witness. (see Sec. 4,
Rule 130, Rules of Court)
168. A orally sold to B his land.
Later B wanted to have the land
registered,
but
registration
requires a public instrument. May
B compel A to execute the
needed public instrument?
It depends. If the contract is
still executor, B cannot compel A to
execute the notarized sale because
the
contract
is
unenforceable.
However, if the price has been paid,
or the land has been delivered, this
time, B can compel A because the
contract
is
both
valid
and
enforceable.
QUESTIONS 169-170
based on the following
situation:

are
fact

Among the items inherited


by B from his mother were some
old paintings that had been
stored in his mothers attic for a
number of years. B knew nothing
about art and had no place to put
them in his house. He placed an

ad in the paper offering to sell


the paintings at a price to be
mutually agreed upon. C, a
buyer
for
an
art
gallery,
responded
to
the
ad
and
examined the paintings. From the
signature
and
the style, C
recognized that the artist was
BenCab, a renowned Filipino
portrait artist. A and B agreed
upon a price and signed a
contract.
169. Which of the following
facts, if true, would give B the
best basis for avoiding the
contract with C?
a. B told C that his mother
had
been
dabbled
in
painting when she was
younger
and
had
undoubtedly painted them
herself.
b. B did not know that C
was a buyer for an art
gallery
and
was
very
familiar with the works of
renowned Filipino artists.
c. B told C that he wanted
to get rid of the paintings
as soon as possible because
he was angry at his mother
for giving away most of her
possessions to her friend
just before he died.
d. C falsely told B that the
paintings were to be used
to
furnish
Cs
newlyconstructed vacation house
in Tagaytay City.
The statement in (A) would
enable B to annul the contract on the
ground of mistake if C was aware that
B was mistaken about the identity of

I press toward
BAR OPERATIONS
2013 the mark for the

prize of the high calling of God


in Christ Jesus. - Philippians
3:14

BARRISTERS CLUB

Page 54 of 67

in CIVIL LAW
Prepared by: Atty. Roney Jone P. Gandeza
the artist. Under the facts in this
choice, C knows that B is mistaken
about the identity of the artist, which
mistake refers to the substance of the
thing which is the object of the
contract.
The statement in (B) is wrong
because the fact that one of the
parties to the contract has superior
knowledge about the subject matter
of the contract does not by itself
justify annulment, even of the other
party is unaware of that fact. Cs
knowledge or lack of it was not the
principal cause on which the contract
was made and was not relied on by B
in making the sale.
The statement in (C) is wrong
because the fact that B was angry
when he agreed to the contract is not
a ground for annulment of a contract
under the law. Regardless of Bs state
of mind, there was a meeting of the
minds between the parties.
The statement in (D) is
incorrect
because
Cs
misrepresentations to B as to how she
will use the paintings does not appear
to go to the substance of the thing
which is the subject matter of the
contract or to have been relied on by
B. Hence, the misrepresentation is
not significant enough to serve as a
ground for annulling the contract.
170. Which of the following
facts, if true, would give C the
best basis for annulling the
contract?
a. Several of the paintings
cracked when they were
being transported by V
because they were brittle
with age.

b.
The
day
after
the
purchase, a respected art
historian announced in a
press release that several
of
Xs
paintings
were
actually
done
by
his
students, causing the value
of all Xs paintings to
decline.
c.
Because
of
some
experimental pigments that
the artist had used, the
colors began to fade rapidly
as soon as the paintings
were exposed to light;
within a few days, virtually
all of the colors had faded
away.
d. The gallery for which C
had procured the paintings
was destroyed by a fire
shortly after the contract
was executed.
(C) offers the best ground for
annulling the contract based on
mutual mistake. When both parties
entering into a contract are mistaken
about facts relating to the agreement,
the contract may be avoidable by the
adversely affected party if the latter
did
not
assume
the
risk
or
contingency of mistake. Here, both
parties believed that the paintings
would be suitable for viewing and had
no reason to suspect that their color
would fade away when exposed to
light. Despite Cs superior knowledge
of the subject matter of the contract,
it is doubtful that she would be
deemed to have assumed the risk or
contingency of what occurred to the
paintings.
The circumstances in (A) are not
that strong as basis for annulment.

I press toward
BAR OPERATIONS
2013 the mark for the

prize of the high calling of God


in Christ Jesus. - Philippians
3:14

BARRISTERS CLUB

Page 55 of 67

in CIVIL LAW
Prepared by: Atty. Roney Jone P. Gandeza
Even
assuming that both parties
mistakenly
believed
that
the
paintings were not too fragile to be
transported, that risk is more likely to
be assumed by C.

The preterition of C annuls the


institution of A, B and F as Ts heirs.
Intestacy results. Therefore, A, B and
C will each get P30, 000. The friend,
F, get nothing.

The circumstances in (B)


suggest only that C has made a
mistake as to the value of the
paintings, but since B knew nothing
about the identify of the artist, the
mistake is unilateral and C cannot
annual on this ground.

173. Testator
T
has
three
legitimate children: A, B and C. In
his will, T disinherits A and
institutes B and C as his heirs.
The disinheritance of A is invalid
because it is for a cause not
provided by the law. If the
hereditary estate is P90, 000,
how shall the distribution be
made?

Choice (D) is incorrect because


the circumstances do not satisfy the
requirements for annulment. Under
Article 1351 of the Civil Code, the
particular motives of the parties in
entering into the contract are
different from the cause thereof.
171. W, wife of H, institutes a
petition
seeking
a
judicial
declaration of presumptive death
of her husband who has been
missing since 2005. Will the
petition prosper?
It depends. If the action is to
enable W to contract a subsequent
marriage, the petition is open the
estate of H to succession or for other
purpose, the petition will not prosper
because the presumption is already
established by law.
172. Testator
T
has
three
legitimate children, A, B and C. T
made a will instituting his
children, A and B, and a friend, F,
as
his
sole
heirs.
C
was
completely
omitted
in
the
inheritance. If Ts estate at the
time of his death is P90, 000,
how should the distribution be
made?

The institution of B and C


remains valid, but their shares are to
be reduced to give A his legitime. Had
there been preterion here, each
would receive P30, 000 each.
Therefore, A is entitled to his legitime
of P15, 000; B to his legitime of P15,
000 plus his share as an instituted
heir in the amount of P22, 500; and C
is entitled to the same share as B.
174. X borrowed from his friend,
Y, the latters car for an out of
town trip. On his way to his
destination, X met with an
accident which greatly damages
the car. X was not at fault
because he was driving carefully.
If the car is to be repaired, the
expenses for repair shall be
borne by:
a. X alone, whether he is at
fault or not.
b. Y alone, because X was
not at fault.
c. Y alone, he being the
owner of the car.

I press toward
BAR OPERATIONS
2013 the mark for the

prize of the high calling of God


in Christ Jesus. - Philippians
3:14

BARRISTERS CLUB

Page 56 of 67

in CIVIL LAW
Prepared by: Atty. Roney Jone P. Gandeza
d. X any Y in equal shares,
the expenses being deemed
extraordinary.
The choice in (D) is correct
because the expenses are deemed
ordinary, and not due to the wear and
tear of the thing. This is pursuant to
the second paragraph of Article 1949
of the Civil Code which provides that
if the extra-ordinary expenses arise
from the actual use of the bailee of
the thing loaned, even though he
acted without fault, the expense is
borne equally by the bailor and the
bailee share and share alike.
175. A
borrowed
Bs
truck.
During a fire which broke out in
As garage, he had time to save
only one vehicle, and he saved
his car instead of the truck. Is A
liable for the loss of Bs truck?
Yes. The baille in commodatum
is liable for the loss of the thing
loaned, even if it should be through a
furtuitious event, if being able to save
either the thing borrowed or his own
thing, he chose to save the latter.
(Art. 1492, CC)
176. D borrowed money from C.
To guarantee payment, D left the
Torrens title of his land to C for
the latter to hold until payment
of the loan. Is there a:
a) contract of pledge?

CC) If at all, there was a pledge of the


paper or document constituting the
Torrens Title, as a movable by itself,
but not of the land which the title
represents. There is no mortgage
because no deed or contract was
executed in the manner required by
law for a mortgage. There is no
contract of antichresis because no
right to the fruits of the property was
given to the creditor. (Art. 2131, CC)
177. A leased a building to B for
a
term
of
ten
years.
B
established a shoe factory in the
building and installed therein
certain machineries. Are the
machineries
movables
or
immovables?
The machineries are movables.
Machineries are deemed immovables
only if:
a) they are placed by the
owner of the tenement.
b) an industry or works is
carried on in the tenement.
c) machinery is intended for
such industry or works.
d) machinery tends directly to
meet the needs of such
industry or works.
In the problem presented, the
machineries are movables since they
were installed by B, and not by A.

b) contract of mortgage?
c) contract of antichresis?
d) none of the above?
None of the above. There is no
pledge
because
only
movable
property may be pledged. (Art. 2094,

178.
Suppose
the
lessee
promised
to
leave
the
machineries to the lessor at the
end of the lease, would that
make any difference in your
answer?

I press toward
BAR OPERATIONS
2013 the mark for the

prize of the high calling of God


in Christ Jesus. - Philippians
3:14

BARRISTERS CLUB

Page 57 of 67

in CIVIL LAW
Prepared by: Atty. Roney Jone P. Gandeza
Yes, because the machineries
would
then
be
considered
as
immovables, because the tenant
would then be considered as the
agent of the lessor.
179. A is the owner of a piece of
land upon which fruits were
grown, raised, harvested, and
gathered by B in bad faith. Who
owns the fruits?
A, the landowner, owns the
fruits with no obligation to indemnify
B, except the latters expense in the
production,
gathering
and
preservation of the fruits. This is
pursuant to the rule that no one may
unjustly enrich himself at the expense
of another. (Art. 442, CC)
ARTICLE 449:
crops have not yet
(here the landowner
without indemnity by
accession continua)

applies if the
been gathered
gets the fruits
the principle of

ARTICLE 443: applies when the


crops have already been gathered.
Article 443 does not apply when the
planter is in good faith, because
under Article 544, he is already
entitled to the fruits received and
there is no necessity of reimbursing
him.
180. X, who was abroad, phoned
his brother, Y, authorizing him to
sell Xs property in Pasay. X sent
the title to Y by courier service.
Acting for his brother, Y executed
a notarized deed of absolute sale
of the land to Z after receiving
payment. The sale is:
a) valid, if the sale was
notarized and buyer paid in full.

b) void, because X should


have authorized Y in writing to
sell the land.
c) valid, because Y was his
brother Xs agent and entrusted
with the title needed to effect the
sale.
d) enforceable, because the
buyer could file an action to
compel X to execute a deed of
sale.
(B) is correct because of the
requirement that the sale of realty
through an agent must be in writing
to be valid. In the problem presented,
the authority given by X to Y was not
in writing; hence, the sale is void.
181. Paterno gave a power of
attorney to Ambrosio authorizing
him to sell Paternos car for
Php200, 000 payable in cash.
Under these circumstances, what
are
the
consequences
of
Ambrosios
appointment
by
Paterno as the latters agent to
sell his car?
The authority of Ambrosio to
sell the car is express. It includes the
implied authority to receive payment
and to give a receipt as they are acts
necessary to accomplish the purpose
of the agency. They are both actual
authority.
If Paterno privately instructed
Ambrosio not to consummate the
sale, the sale by Ambrosio is still
binding upon Paterno as Ambrosio
had apparent or ostensible authority
to sell. As far as third persons are
concerned, the effect is as if
Ambrosio had actual authority.

I press toward
BAR OPERATIONS
2013 the mark for the

prize of the high calling of God


in Christ Jesus. - Philippians
3:14

BARRISTERS CLUB

Page 58 of 67

in CIVIL LAW
Prepared by: Atty. Roney Jone P. Gandeza
The same is true if Paterno had
not authorized Ambrosio to sell the
car but having knowledge that
Ambrosio was acting for him kept
silent and after consummation of the
sale, received the proceeds thereof
from Ambrosio. Here, Ambrosios
authority rests on estoppels on the
part of Paterno to deny such
authority.
The
authority
given
to
Ambrosio to sell the car is special
because it involves a particular
transaction.
Ambrosio
has
no
authority to use the car for his own
purposes but he can use it in an
emergency as, for example, to take a
member of his family to a hospital. In
this case, his authority is demanded
by necessity.
182. Give examples of agency by
estoppels and implied agency.
AGENCY BY ESTOPPEL: P
tells X that A is authorized to sell
certain merchandise. P privately
instructs A not to consummate the
sale but merely to find out the
highest price X is willing to pay for
the merchandise. If A makes a sale to
X, the sale is binding on P who is in
estoppels to deny As authority. In
this case, there is no agency created
but there is a power created in A to
create contractual relations between
P and a third person, without having
authority to do so. The legal result is
the same as if A had authority to sell.
IMPLIED
AGENCY:
P
authorized A to sell his car. A sold the
car to X who paid the purchase price.
However, A did not give the money to
P. X is not liable to P. A has implied
authority to receive payment.

183. Under Article 1772 of the


Civil Code, every contract of
partnership having a capital of 3,
000 or more, in money or
property, shall appear in a public
instrument,
which
must
be
recorded in the office of the
Securities
and
Exchange
Commission.
Suppose
this
requirement
has
not
been
complied with, is the partnership
still a juridical person, assuming
that all other requisites are
present?
Yes. Article 1768 of the Civil
Code provides that the partnership
has a juridical personality separate
and distinct from that of each of the
partners, even in case of failure to
comply with the requirements of
Article 1772.
It
is
settled
that
the
registration under Article 1772 of the
Civil Code is not intended as a
prerequisite for the acquisition of
juridical
personality
by
the
partnership, but merely as a condition
for the issuance of the license to
engage in business or trade.
184. X, Y and Z entered into
partnership under the firm name
XYZ
Ltd.
Upon
mutual
agreement, X withdrew from the
partnership and the partnership
was dissolved. However, the
remaining partners, Y and Z, did
not
terminate
the
business.
Instead
of
winding
up
the
business of the partnership and
liquidating its assets, Y and Z
continued the business in the
name of XYZ Ltd. Apparently
without the objection of X. The
withdrawal
of
X
from
the
partnership was not published in
the papers.

I press toward
BAR OPERATIONS
2013 the mark for the

prize of the high calling of God


in Christ Jesus. - Philippians
3:14

BARRISTERS CLUB

Page 59 of 67

in CIVIL LAW
Prepared by: Atty. Roney Jone P. Gandeza
Is X liable for any obligation
Y and Z might incur while doing
business in the name of XYZ
Ltd. after his withdrawal from
the partnership?
Yes, X can be held liable under
the doctrine of estoppels. But as
regards
the
parties
among
themselves, only Y and Z are liable. X
can be held liable since there was no
proper notification or publication to
the public in general. In the event
that X is made to pay the liability to a
third person, he has the right to seek
reimbursement from Y and Z. (Arts.
1837 to 1849, CC)
185. A, B and C are general
partners in s trading frim. Having
contributed equal amounts to the
capital, they agreed on equal
distribution of whatever net
profile is realized per fiscal
period. After four years of
operation, A conveyed his whole
interest in the partnership to D
without
the
knowledge
and
consent of B and C. Is the
partnership dissolved?
No. Under Article 1813 of the
Civil Code, the conveyance by a
partner of his or her whole interest in
the partnership does not itself
dissolve the partnership on the
absence of an agreement.
186. What are the rights of D
shoule he desire to participate in
the
management
of
the
partnership
and
in
the
distribution of the net profit of
P45, 000 which was earned after
his purchase of As interest?

participate in the management or


administration of the partnership
business affairs. He may, however,
receive the net profits to which A
would have otherwise been entitled.
In This case, D is entitled to 1/3 of the
profits.
187. Due to a misunderstanding,
Atty.
Dumag
withdraws
his
membership from a law firm
which does not have a fized term.
He later files with the SEC a
petition for the dissolution of the
firm. After due proceedings, the
SEC hearing officer denied the
petition on the ground that the
withdrawal of Atty. Dumag has
not dissolved the firm
Is the SEC hearing officer
correct?
No. A partnership which does
not have a fixed term is a partnership
at will. The law firm Atty. Dumag is
one such partnership. The birth and
life of a partnership at will is
predicated on the mutual desire and
consent of the partners.
The right to choose with whom
a person wishes to associate himself
is the very foundation and essence of
that
partnership.
Its
continued
existence is, in turn, dependent on
the constancy of that mutual resolve,
along with each partners capability
to give it. Verily, anyone of the
partners may, at his sole pleasure,
dictate dissolution of the partnership
at will. He must, however, act in good
faith. If he acted in bad faith, it would
not prevent the dissolution of the
partnership but that it can result in a
liability for damages. (Ortega, et.al.,
v. Court of Appeals, 245 SCRA 529
[1995])

Also pursuant to Article 1813 of


the Civil Code, D cannot interfere nor

I press toward
BAR OPERATIONS
2013 the mark for the

prize of the high calling of God


in Christ Jesus. - Philippians
3:14

BARRISTERS CLUB

Page 60 of 67

in CIVIL LAW
Prepared by: Atty. Roney Jone P. Gandeza
188. A, B and C are partners in a
construction firm. Due to a
quarrel, C withdrew from the
partnership in 1986 as a result of
which they agreed to dissolve
their partnership and executes
an agreement of partition and
distribution of the partnership
properties. In 1994, Cs heirs
brought an action against A and
B
for
accounting
of
the
partnership assets and partition.
A and B defended on the ground
of prescription. They contend
that the action had already
prescribed four years after it
accrued
in 1986
when
the
partnership was dissolved by the
withdrawal of C. Has the action
prescribed?
Contrary
to
A
and
Bs
allegation, prescription has not even
begun to run in the absence of a final
accounting. Article 1842 of the Civil
Code states that the right to demand
accounting accrues at the date of
dissolution in the absence of any
agreement to the contrary. When a
final accounting has been made, it is
only then that prescription begins to
run.
In the case at bar, no final
accounting has been made, and that
is precisely what the heirs of C are
seeking since A and B failed or
refused to render an accounting of
the
partnerships
business
and
assets. Hence, the action is not
barred by prescription. (Emnace v.
Ca, 370 SCRA 431)
189. C loans D the amount of
P100, 000. As security for the
loan, D delivered to C two rings
by way of pledge. When D failed
to pay, C foreclosed, and had the
rings
sold
at
auction.
The

proceeds of the sale, after


deducting expenses, amounted
only to P70, 000.
Is C entitled to demand the
deficiency from D?
No. The Civil Code on pledge
provides that the foreclosure of the
pledge extinguishes the principal,
whether or not the proceeds of the
sale are more, or less than the
amount of the principal obligation.
(Art. 2115, CC)
Under Article 2115 of the Civil
Code, if the price of the sale is less
than the principal obligation, the
creditor shall not be entitled to
recover
the
deficiency,
notwithstanding any stipulation to the
contrary.
190. What if the rings were sold
for the net amount of P150, 000,
is D entitled to the excess?
Under Article 2115 of the Civil
Code, if the price of the sale is more
than the amount of the principal
obligation, the debtor shall not be
entitled to the excess, unless it is
otherwise agreed upon. (Art. 2115,
CC)
191. Suppose the rings had been
chattel mortgaged to C, is D
liable to pay the deficiency if the
proceeds of the sale were less
than the indebtedness? Or, is she
entitled to the excess, if the
proceeds were more?
If instead of a pledge the rings
were given by way of a chattel
mortgage,
C
can
recover
the
deficiency from D as there is no
prohibition in the Chattel Mortgage
Law similar to pledge. The excess, if

I press toward
BAR OPERATIONS
2013 the mark for the

prize of the high calling of God


in Christ Jesus. - Philippians
3:14

BARRISTERS CLUB

Page 61 of 67

in CIVIL LAW
Prepared by: Atty. Roney Jone P. Gandeza
any, should be returned to D, the
mortgagor.
192. Lessor and Lessee verbally
agree that if Lessor should
decide to sell the building, he will
give Lessee the pre-emptive right
to buy the leased property. The
following year, Lessor offered to
sell to Lessee the building for P2
million,
but
Lessee
counteroffered for P1 million.
Lessee later learned that a
buyer had already purchased the
property from Lessor for P800,
000. He also discovered that the
sale
had
already
been
consummated
when
Lessor
offered to sell it to him. Lessee
thus offered to reimburse the
buyer the purchase price of P800,
000, plus an additional P200, 000
to complete his earlier offer of P1
million.
When
the
offer
was
refused,
Lessee
brought
an
action for rescission of the sale.
After due proceedings, however,
the
court
dismissed
the
complaint on the ground that
the right of redemption on
which the complaint is based is
merely an oral one as such, is
unenforceable under the law.
Is the right
governed by the
Frauds?

of refusal
Statute of

No. A right of first refusal is not


among those listed as unenforceable
under the Statute of Frauds.
The application on Article
1403, par. 2(e) of the Civil Code
presupposes the existence of a
perfected contract of sale of real

property. It is a contractual grant, not


of the sale of real property involved
but of the right of first refusal over
the property sought to be sold. The
right of first refusal need not be
written to be enforceable and may br
proved by oral evidence. (Rosencor
Devt. V. Inquing, 354 SCRA 119)
193. T died in 1990 with a will.
In his will, he devised a house
and lot to his friend, A, as first
heir and to C, As son, as second
heir. B died in 1995 survived by
his two children E and F. A
himself died in 2000 survived by
his children C and D.
In the settlement of As
estate, E and F filed a motion to
exclude
the house and
lot
originating from T on the ground
that they are the exclusive
owners of the property. C and D
opposed the motion on the
ground that B, the second heir,
predeceased
A,
and
that
therefore, the fideicommissary
substitution did not produce any
effect as far as B, the second
heir, is concerned.
Should the opposition be
sustained?
No. B, the second heir,
acquires a right to the succession
from the time of the testators death,
even though he, B, should die before
the fiduciary, A.
B inherited from T as second
heir when the latter died in 1990.
When B died in 1995, he was able to
transmit his right to his own heirs, E
and F.

I press toward
BAR OPERATIONS
2013 the mark for the

prize of the high calling of God


in Christ Jesus. - Philippians
3:14

BARRISTERS CLUB

Page 62 of 67

in CIVIL LAW
Prepared by: Atty. Roney Jone P. Gandeza
When A (first heir) died in
2000, the right of E and F over the
property became absolute.
194. H died leaving an estate of
P100, 000. His widow, W, gave
birth to a child four months after
Hs death, but the child died five
hours after birth. Two days after
the childs death, W also died
because she had suffered from
difficult child birth. The estate of
H is now being claimed by his
parents, A and B, and by C and D,
the parents of W. Who is entitled
to Hs estate of P100, 000?
If the child had an intra-uterine
life of not less than seven months, it
inherited
from
the
father.
Consequently, the estate of P100,
000 shall be divided equally between
the child and his mother as legal
heirs. Upon the death of the child, its
share of P50, 000 goes by operation
of law to the mother, W, which is
subject to reserve troncal.
Under Article 891 of the Civil
Code, the reserve is in favor of
relatives belonging to the paternal
line and who are three degrees from
the child. The parents of H (A and B)
are entitled to the reserved portion
which is P50, 000 as they are two
degrees related from the child. The
P50, 000 inherited by W from H will
go to her parents, C and D, as her
legal heirs.
However, if the child had an
intra-uterine life of less than seven
months, half of the estate of H, or
P50, 000, will be inherited by W, the
widow while the other half, P50, 000,
will be inherited by the parents of H.
Upon the death of W, her estate of
P50, 000 will be inherited by her own
parents, C and D.

195. Before his death in 1990, A


donated to his grandson, F, a
child of his predeceased son D, a
house and lot worth P600, 000.
In 1995, F died with a will
instituting his mother, E, as his
sole heir. His estate consisted
entirely of the house and lot
which he had received from A.
In 1998, E also died but
without a will. The house and lot
is now claimed by: (a) B, widow
of A and grandmother of F; (b) C,
son of A and B uncle of F; and (c)
G, sister and only living relative
of E. To who, shall the property
be adjudicated?
Half to C; other half to G.
F, the propositus, died with a
will instituting his mother, E, as sole
heir.
Consequently, only one0half of
the property passed to her by
operation of law since that is her
legitime. Only this portion of the
property has become reservable.
B is not entitled to the
reservable portion of the property.
Although a relative of the
propositus in the second degree, B is
merely related by affinity to the
ascendant (A) from whom the
property came. She does not,
therefore, belong to the line from
which the property came.
A reservatario must not only be
related by consanguinity to the
propositus within the third degree,
but he must also be related by

I press toward
BAR OPERATIONS
2013 the mark for the

prize of the high calling of God


in Christ Jesus. - Philippians
3:14

BARRISTERS CLUB

Page 63 of 67

in CIVIL LAW
Prepared by: Atty. Roney Jone P. Gandeza
consanguinity to the ascendant from
whom the property came.
C is entitled to the reservable
portion of the property since he is not
only a third degree relative by
consanguinity of the propositus, but
he also belongs to the line from which
the property came.

FIRST, the property had been


acquired by operation of law by an
ascendant (F)from his descendant (Y)
upon the death of the latter.
SECOND, the property had
been
previously
acquired
by
gratuitous title by the descendant (Y)
from a brother (X).

G, on the other hand, is


entitled to the part of the property
which is not reservable in accordance
with the ordinary rules of intestate
succession.
196. D, only daughter of B,
married E, only son of A, in 1981.
A son, X, was born to the couple
in 1982. E died in a vehicular
accident in 1984. In 1986, D
married F, only son of C. A son, Y
was born to the couple in 1988. D
also died in a vehicular accident
in 1992. In 1995, X, who was very
sickly,
donated
to
his
half
brother, Y, a parcel of land. X
died the following year. In 1998 Y
also died. He died intestate and
without any surviving issue. The
land which he had acquired from
X was inherited by his father, F,
who was his only legal heir. F
died intestate in 2002, survived
only by his father, C. The land
which had originated from X is
now being claimed by A and B on
the ground that it is reservable.
C claims that the property
belongs to him and him alone as
his inheritance from his son, F.

THIRD, descendant (Y) died


without any legitime issue who can
inherit from him.

Who
among
the
grandfathers is entitled to the
property?

EXCEPT
FOR
CHATTEL
MORTGAGE, a pledge, real estate
mortgage,
or
antichresis
may
exceptionally secure after-incurred
obligations so long as these future
debts are accurately described. This
is so because chattel mortgages can

B alone is entitled to the


property. The property is reservable.

In order to determine who can


qualify as a reservatario, two test
should be applied: First is the
claimant a relative of the descendantpropositus within the third degree?
Second, does he belong to the line
from which the reservable property
came?
Applying the tests, it is clear
that:
Neither can C qualify because
he does not belong to the line from
which the property came. He is not
related by consanguinity to X.
Only B can qualify. He is not a
relative of Y within the third degree;
he also belongs to the line from which
the reservable property came.
197. May a contract of pledge,
real estate mortgage, antichresis
and chattle mortgage secure
after-incurred obligation?

I press toward
BAR OPERATIONS
2013 the mark for the

prize of the high calling of God


in Christ Jesus. - Philippians
3:14

BARRISTERS CLUB

Page 64 of 67

in CIVIL LAW
Prepared by: Atty. Roney Jone P. Gandeza
only cover obligations existing at the
time the mortgage is constituted.
One of the requirements of
chattel mortgage is an affidavit of
good faith and the law has provided
that the parties to the contract must
execute an oath that the mortgage is
made for the purpose of securing the
obligation specified in the conditions
thereof and for no other purposes.
The debt referred to in the law is a
current obligation, not an obligation
that is merely contemplated.
198. Which
irregularities
on
formal requisites of marriage
would not affect the validity of a
marriage but may render the
party or parties civilly, criminally,
or administratively liable?
a) Marriage license is issued by
a local civil registrar of a city or
municipality where neither party
habitually resides.
b) Marriage license is presigned by a local civil registrar but
issued by a fixer.
c) Marriage is performed at a
place other than those designated by
law without the written request under
oath of the parties.
d) No marriage counseling
certificate
is
attached
to
the
application and the marriage license
is issued even before the lapse of the
three0month period, the marriage is
valid.
e)
Marriage
ceremony
is
performed with only one witness
present or with no witnesses at all
(secret marriage).

f) Marriage license is issued


without waiting for the lapse of the
three-month period.
g) Marriage license is issued
without waiting for the lapse of the
ten-day period, or if the notice of
application is not published.
h) Marriage remains valid
neither contracting party belongs to
the solemnizing officers church or
religious sect.
i) The solemnizing officer is not
duly registered with the civil registrar
general at the time of the marriage.
j) The solemnizing officer
solemnized the marriage in violation
of the written authority granted to
him by his church or religious sect,
the marriage is valid.
k) A judge
marriage
outside
jurisdiction.

solemnizes a
his
courts

l) Alien spouse obtains a


marriage license without a certificate
of legal capacity.
199. If
a
party
has
been
previously married, and the prior
marriage has been dissolved, is
parental consent still required if
such party is still below 21 years
of age?
Not anymore. This is so
because of Article 14 of the Family
Code which provides, In case either
or both of the contract parties, not
having been previously emancipated
by a previous marriage, x x x. This
means parental consent is no longer
required for these parties.

I press toward
BAR OPERATIONS
2013 the mark for the

prize of the high calling of God


in Christ Jesus. - Philippians
3:14

BARRISTERS CLUB

Page 65 of 67

in CIVIL LAW
Prepared by: Atty. Roney Jone P. Gandeza
200. What is
possession?

mortgage

in

A mortgage in possession,
otherwise known as antichresis, is
one where the mortgagee acquires
actual or constructive possession of
the property mortgaged for purposes
only of enforcing his security over the
property and collecting the income to
pay for the mortgagors debt. (Nadal
v. CA, 320 SCRA 699)

unless such usufruct has been


granted only in consideration of the
existence of such person.
If the document constituting V
as a usufructuary does not state that
it will end the moment Mrs. X is dead,
then it will continue.

201. A and B are childhood


friends.
Because
of
their
friendship, A gave to B in
usufruct a parcel of land to last
up to the time their high school
teacher, Mrs. X, reaches the age
of 70. Mrs X died at the age of
65. Is the usufruct extinguished?
No. Under Article 606 of the
Civil Code, a usufruct granted for a
time that may elapse before a third
person attains a certain age shall
subsist from the number of years
specified even if the third person
should die before the period expires,

I press toward
BAR OPERATIONS
2013 the mark for the

prize of the high calling of God


in Christ Jesus. - Philippians
3:14

BARRISTERS CLUB

Page 66 of 67

THE BARRISTERS CLUB


OFFICERS
Chancellor:
Vice Chancellor:

ABBYGAILE T. GONZALES
ROMEL L. BASILAN

Secretary:

JESSA ALYSSA G. REYES

Treasurer:

MILDRED P. AMBROS

PRO:

ROBYN B. DELA PENA

PRO:

AARON JAMES E. CO

Business Manager:

RUDDY ALLEN N. YEE

Business Manager:

LESLIE D. RAGUINDIN

SSG
Representative:
Ex-Officio:

ANNE LUCILLE B. RUIZ

Adviser:
Dean, College of
Law:

RONA B. ESTRADA
ATTY. ISAGANI G. CALDERON
ATTY. REYNALDO U.
AGRANZAMENDEZ

Potrebbero piacerti anche